Nainital Bank PO & SO - SPLessons

Nainital Bank Recruitment Model Paper – PO & SO

Home > > Tutorial
SPLessons 5 Steps, 3 Clicks
5 Steps - 3 Clicks

Nainital Bank Recruitment Model Paper – PO & SO

shape Introduction

Aspirants of Nainital Bank Recruitment 2019 Notification, who are willing to enhance their career with banking sector can check all details in Nainital Bank Recruitment Notification - PO & SO regarding filling of PO & SO Job Vacancies. The Bank organization invites online application from eligible candidates having B.Tech, M.Sc, B.Sc, MCA, MBA, CA, Post Graduate qualifications. These 130 PO & SO Posts are in Nainital Bank, Nainital, Uttarakhand. The article Nainital Bank Recruitment Model Paper - PO & SO provides pattern Questions, and it helps the candidates to devise effective preparation strategies, by focusing on their strengths and weaknesses in different topics and also useful to the candidates while preparing Various Insurance, Banking & Government Exams like SBI PO, SBI Clerk, SSC CPO, SSC CHSL.

shape Pattern

Sr. No. Name of the Test No. of Question Maximum Marks Version Duration
1. Reasoning 60 60



Only English
45 Minutes
2. English Language 40 40 35 Minutes
3. Quantitative Aptitude 50 50 40 Minutes
4. General Awareness (with special reference to Banking) 50 50 25 Minutes
Total 200 200 145 Minutes
Negative Marking- Penalty of 0.25 marks for each wrong answer

shape Sample

1. Which Bank has launched its Multi Option Payment Acceptance Device (MOPAD) for digital payments
    A. Axis Bank B. State Bank of India C. Punjab National Bank D. ICICI Bank E. Bank of Baroda

Answer Option: B
Explanation: State Bank of India, the largest lender with a fifth of the market share, launched a payments machine that would help merchants eliminate the multiple choices that they keep to facilitate transactions from cards to QR code-based payments. The new device titled MOPAD, or Multi Option Payment Acceptance Device, is a Point of Sale (PoS) terminal that would along with cards accept payments through UPI, Bharat QR, and SBI Buddy wallet which till now required different tools to receive payments.
2. The Reserve Bank of India has constituted a 10-member ‘High Level Task Force on Public Credit Registry (PCR) for India’, which will, among other things, suggest a roadmap for developing a transparent, comprehensive and near-real-time PCR for India. Who is the head of Public Credit Registry?
    A. Sekar Karnam B. Rashesh Shah C. Vishaka Mulye D. Sriram Kalyanaraman E. YM Deosthalee

Answer Option: E
Explanation: The Reserve Bank of India has constituted a 10-member ‘ High-Level Task Force on Public Credit Registry (PCR) for India’, which will, among other things, suggest a roadmap for developing a transparent, comprehensive and near-real-time PCR for India. Headed by YM Deosthalee, ex-CMD, L&T Finance Holdings, the task force includes Sekar Karnam, DMD & Chief Credit Officer, SBI; Vishaka Mulye, ED, ICICI Bank; Rashesh Shah, Chairman, and CEO, Edelweiss Group; and Sriram Kalyanaraman, MD and CEO, National Housing Bank.
3. Kharchi puja is a Hindu festival of which state?
    A. Tripura B. Manipur C. Assam D. Sikkim E. Meghalaya

Answer Option: A
Explanation: Kharchi puja is a Hindu festival from Tripura, India. Performed in Agartala in July or August, the festival involves the worship of the fourteen gods forming the dynasty deity of the Tripuri people. Kharchi Puja is one of the most popular festivals in Tripura.
4. India’s first government run e-waste recycling plant has opened in which city?
    A. Mumbai B. Hyderabad C. New Delhi D. Bengaluru E. Kolkata

Answer Option: D
Explanation: Bengaluru, the IT capital of India, is also the third in terms of e-waste generation in the country. While India produces 18.5 lakh metric tonnes of electronic waste annually,?a whopping 92,000 metric tonnes comes from Bengaluru, according to 2016 figures. And as per the study, computers form 70% of the total e-waste and telecommunication equipment constitutes 12%. The Centre, noting that Bengaluru is one of the topmost e-waste producers nationally, has decided to sanction India’s first government-run e-waste recycling unit.
5. Where is the headquarter of Karur Vysya Bank?
    A. Bengaluru, Karnataka B. Thrissur, Kerala C. Karur, Tamil Nadu D. Jaipur, Rajasthan E. Kochi, Kerala

Answer Option: C
Explanation: Karur Vysya Bank (Tamil) is an Indian old private-sector bank, headquartered in Karur in Tamil Nadu.
6. "Sulabh Jal Scheme" has been launched in which state?
    A. Indore, Madhya Pradesh B. Jodhpur, Rajasthan C. Rajkot, Gujarat D. Guwahati, Assam E. Darbhanga, Bihar

Answer Option: E
Explanation: An innovative cost-effective drinking water project in Bihar promises to lower the price of one-litre bottle to 50 paise (cheapest in the world). The project- "Sulabh Jal" was launched in Darbhanga by Sulabh International.
7. Which of the following state has bagged the top position under the Pradhan Mantri Surakshit Matritav Abhiyan (PMSMA)?
    A. Andhra Pradesh B. Madhya Pradesh C. Uttar Pradesh D. Himachal Pradesh E. Arunachal Pradesh

Answer Option: D
Explanation: Himachal Pradesh has been adjudged first among states for its performance under the Pradhan Mantri Surakshit Matritav Abhiyan (PMSMA) in the country. Union Health Minister Jagat Prakash Nadda conferred the award upon the state which was received by Additional Chief Secretary, Health, BK Agarwal at a ceremony in New Delhi.
8. Nelson Mandela Foundation is dedicating this year's (2018) Mandela Day to Action Against-
    A. Pollution B. Corruption C. Poverty D. Global Warming E. Polio

Answer Option: C
Explanation: One hundred years after his birth, Mandela's example of courage and compassion continue to inspire the world. The Nelson Mandela Foundation is dedicating this year's (2018) Mandela Day to Action Against Poverty, honouring Mandela's leadership and devotion to fighting poverty and promoting social justice for all.
9. What is the allocation on Digital India Project under Union budget 2018-19?
    A. Rs. 1014 crore B. Rs. 3073 crore C. Rs. 5054 crore D. Rs. 2315 crore E. Rs. 9432 crore

Answer Option: B
Explanation: In his address to the Parliament, while presenting the Budget for 2018, Finance Minister Arun Jaitley stated that NITI Aayog will initiate a national program to direct efforts in artificial intelligence. The Budget doubled the allocation on Digital India programme to Rs. 3073 crore in 2018-19.
10. The 10th edition of the Defence Expo was held in-
    A. Chennai B. Kolkata C. New Delhi D. Mumbai E. Bhopal

Answer Option: A
Explanation: Prime Minister Narendra Modi was inaugurated 10th edition of Defence Expo (DefExpo 2018) in Chennai at Tiruvidantha
11. Which of the following state has launched ‘One Person One Car’ policy?
    A. West Bengal B. Kerala C. Madhya Pradesh D. Rajasthan E. Gujarat

Answer Option: A
Explanation: West Bengal chief minister Mamata Banerjee set a “one person one car” policy for ministers and bureaucrats and mandated economy class for all domestic air travel in a bid to cut costs incurred by the state government.
12. Who has been named as ‘British Indian of the Year’?
    A. Anoushka Shankar B. Ishwar Sharma C. Shahraan Dutt D. Arhaan Khan E. MC Rama Rao

Answer Option: B
Explanation: An eight-year-old Indian-origin schoolboy who is the under-11 UK national yoga champion has been named the British Indian of the Year for his achievements in the field. Ishwar Sharma has won a string of titles in both individual and artistic yoga, most recently a gold medal representing Great Britain at the World Student Games 2018 in Winnipeg, Canada, in June 2018.
13. The Government of India has notified the Electoral Bond Scheme 2018. It may be noted that Electoral Bond shall be valid for _____________ days from the date of issue.
    A. 90 B. 30 C. 15 D. 10 E. 20

Answer Option: C
Explanation: The Government of India has notified the Electoral Bond Scheme 2018. As per provisions of the Scheme, Electoral Bonds may be purchased by a person, who is a citizen of India or incorporated or established in India. It may be noted that Electoral Bond shall be valid for fifteen days from the date of issue and no payment shall be made to any payee Political Party if the Bond is deposited after expiry of the validity period. The bond deposited by any eligible political party to its account shall be credited on the same day
14. Which bank has been ranked as the best PSU bank in the overall digital transaction’s category in India?
    A. Bank of India B. State Bank of India C. Union Bank of India D. Punjab National Bank E. Bank of Baroda

Answer Option: D
Explanation: Punjab National Bank (PNB), which was recently hit by a multi-billion dollar fraud by manipulation of its financial messaging --Swift -- system, has been ranked as the best PSU bank in the overall digital transactions category in India. The bank said in a statement on Saturday: "Based on the recent findings of DFS (Department of Financial Services), PNB is ranked the number 1 PSU bank in digital transactions in India." An Rs. 13,500-crore fraud came to light in January, when it was found that the financial messaging system SWIFT was manipulated. It was used to issue Letters of Undertakings (LoUs) to conduct the fraud.
15. India's first engineless rail, Train 18, manufactured by the Integral Coach Factory (ICF) was rolled out in-
    A. Bhopal B. New Delhi C. Kapurthala D. Madhepura E. Chennai

Answer Option: E
Explanation: India's first engineless rail, Train 18, manufactured by the Integral Coach Factory (ICF) was rolled out on October 29 in Chennai.
16. The Union Cabinet has gave its approval to rename the Agartala Airport in Tripura as?
    A. Om Prakash Gattan, Airport B. Anuradha Devi Thokchom Airport C. Denis P Rayen Airport D. Maharaja Bir Bikram Manikya Kishore Airport E. Pabitra Rabha Dinesh Airport

Answer Option: D
Explanation: The Union Cabinet chaired by the Prime Minister, Shri Narendra Modi has gave its approval to rename the Agartala Airport in Tripura as ‘Maharaja Bir Bikram Manikya Kishore Airport, Agartala. The decision comes in the wake of the long pending demand of people of Tripura as well as the Tripura Government for paying tribute to Maharaja Bir Bikram Manikya Kishore.
17. Which state has launched 'I am not afraid of English' initiative to promote English language right from Class 1 in the state''s primary schools?.
    A. Haryana B. Bihar C. Uttar Pradesh D. Rajasthan E. West Bengal

Answer Option: A
Explanation: The Haryana Education Department has launched ''I am not afraid of English'' initiative to promote the English language right from Class 1 in the state's primary schools. The initiative is aimed at capacity building of teachers to enable them to help the students to learn, read, write and speak English.
18. Microdot technology in India to be launched by the Government of India to check-?
    A. Children thefts B. Petrol thefts C. Vehicle thefts D. Books thefts E. Girls thefts

Answer Option: C
Explanation: Microdot technology in India to be launched by the Government of India to check vehicle thefts
19. Under which ministry the Swachh Bharat Mission (Gramin) and the Swachh Bharat Mission (Urban) has launched?
    A.Ministry of Housing and Urban Affairs B. Ministry of Home Affairs C. Ministry of Human Resource Development D. Ministry of Environment, Forest and Climate Change E.Ministry of Drinking Water and Sanitation

Answer Option: E
Explanation: Ministry of Drinking Water and Sanitation has launched the Swachh Bharat Mission (Gramin) and the Swachh Bharat Mission (Urban).
20. The environment ministry re-introduces a scheme ‘Medini Puraskar Yojna’ which aims at promoting original writing of books in which language?
    A.Tamil B.Hindi C.Urdu D.Bengali E.English

Answer Option: B
Explanation: The environment ministry has decided to re-introduce a scheme "The Medini Puraskar Yojnaa", which aims at promoting original writing of books in Hindi, will have four awards for authors of various environmental topics.
21. Petroleum Minister Dharmendra Pradhan has launched which initiative to promote Compressed Bio-Gas as an alternative, green transport fuel?
    A.SATAT B.MOPAD C.WINGS D.BHARAT E.NIPAN

Answer Option: A
Explanation: Shri Dharmendra Pradhan, Union Minister of Petroleum and Natural Gas & Skill Development and Entrepreneurship has kicked off an innovative initiative in New Delhi on 1st October, 2018, with PSU Oil Marketing Companies (OMCs ,i.e. IOC, BPCL and HPCL) inviting Expression of Interest (EoI) from potential entrepreneurs to set up Compressed Bio-Gas (CBG) production plants and make available CBG in the market for use in automotive fuels. This significant move has the potential to boost the availability of more affordable transport fuels, better use of agricultural residue, cattle dung and municipal solid waste, as well as to provide an additional revenue source to farmers. Titled SATAT, the initiative is aimed at providing a Sustainable Alternative Towards Affordable Transportation (SATAT) as a developmental effort that would benefit both vehicle-users as well as farmers and entrepreneurs.
22. Indian naval ship _______________ has became the first-ever warship to enter port of Sabang in Indonesia.
    A. INS Arihant B. INS Sumitra C. INS Vikrant D. INS Trishul E. INS Garur

Answer Option: B
Explanation: Indian naval ship INS Sumitra became the first-ever warship to enter the port of Sabang in Indonesia. The Indian Navy said INS Sumitra was received by traditional dancers and Indonesian traditional band. The ship was welcomed by Indian Ambassador to Indonesia PK Rawat, Indonesian foreign affairs officials, members from the Indian embassy, Indian businessmen and Indonesian naval and air force officers onboard. The warship was deployed in Malacca Straits.
23. NABARD has joined hands with which bank to provide collateral-free credit through Joint Liability Groups in Telangana?
    A. BOB B. ICICI Bank C. SBI D. Axis Bank E. PNB

Answer Option: C
Explanation: SBI and Nabard tie-up for credit in Telangana. State Bank of India and National Bank for Agriculture and Rural Development have joined hands to provide collateral-free credit through Joint Liability Groups in Telangana.
24. The Government of India has signed a loan deal worth USD 200 million with the World Bank for
    A.APY B.PMKSY C.NIRVAAN D.POSHAN E.ATAL

Answer Option: D
Explanation: The Government of India signed a loan deal worth USD 200 million with the World Bank for the National Nutrition Mission (POSHAN Abhiyaan). The POSHAN Abhiyaan, an overarching scheme for holistic nourishment, was launched by Prime Minister Narendra Modi in March this year at Jhunjhunu, Rajasthan
25. Under project ‘SASHAKT’ an asset management company/ alternative investment fund (AIF)- led resolution approach to deal with NPA cases of more than-?
    A. Rs 100 crore B. Rs 200 crore C. Rs 300 crore D. Rs 400 crore E. Rs 500 crore

Answer Option: E
Explanation: Project "Sashakt" aims to strengthen the credit capacity, credit culture and credit portfolio of public sector banks. The committee has set a five-prong strategy towards resolution of stressed assets. ‘SASHAKT’ stands for strengthening and the whole objective was to strengthen the credit capacity, credit culture and portfolio of public sector banks. The AMC will be set up by state-run banks for resolution of loans above Rs 500 crore.
26. National Mission for Clean Ganga Organises “Ganga Vriksharopan Abhiyan” in Five Ganga Basin States Intervention in Ganga a Major Components of Namami Gange Programme. These 5 states are Uttarakhand, Uttar Pradesh, Bihar, West Bengal and-?
    A. Assam B. Jharkhand C. Himachal Pradesh D. Haryana E. Odisha

Answer Option: B
Explanation: NMCG Organises “Ganga Vriksharopan Abhiyan” in Five Ganga Basin States Intervention in Ganga a Major Components of Namami Gange Programme. National Mission for Clean Ganga (NMCG) is running “Ganga Vriksharopan Abhiyan” in five main stem Ganga basin states – Uttarakhand, Uttar Pradesh, Bihar, Jharkhand and West Bengal.
27. Prime Minister Narendra Modi has launched "Rashtriya Gram Swaraj Abhiyan" at Ramnagar, tribal-dominated district of Madhya Pradesh. The scheme aims to strengthen the country's-
    A. NPA Loss System B. Corruption Based System C. Politics System D. Labour System E. Panchayti Raj System

Answer Option: E
Explanation: Prime Minister Narendra Modi has launched a scheme that seeks to strengthen the country's Panchayati Raj system and address critical gaps that hinder its success. He launched the Rashtriya Gram Swaraj Abhiyan at Ramnagar in this tribal-dominated district of Madhya Pradesh on the occasion of National Panchayati Raj Day.
28. The government was announced a scheme for informants of benami transactions under which an individual can get a reward of up to _____________ for providing information to the tax authorities.
    A. Rs. 1 crore B. Rs. 7 crore C. Rs. 4 crore D. Rs. 10 crore E. Rs. 5 crore

Answer Option: A
Explanation: The government announced a scheme for informants of Benami transactions under which an individual can get a reward of up to ? 1 crore for providing information to the tax authorities. “Under the Benami Transactions Informants Reward Scheme 2018, a person can get reward [of] up to ? 1 crore for giving specific information in a prescribed manner to the Joint or Additional Commissioners of Benami Prohibition Unit (BPU) in Investigation Directorates of Income Tax Department about Benami transactions and properties as well as proceeds from such properties which are actionable under Benami Property Transactions Act, 1988, as amended by Benami Transactions (Prohibition) Amendment Act, 2016.
29. NITI Aayog presented India’s Voluntary National Review Report on the implementation of the
    A. UN Member Countries B.International Review of Progress C. Sustainable development Goals D.All of the above E.None of the given options is true

Answer Option: C
Explanation: As a signatory to the 2030 Agenda for Sustainable Development, India is committed to participating in the international review of the progress of Sustainable development Goals (SDGs) on a regular basis. NITI Aayog presented India’s Voluntary National Review Report on the implementation of the Sustainable development Goals (SDGs)
30. Which Bank has entered into an agreement with CDSL Commodity Repository Limited (CCRL) and it is first Public Sector Bank to become the Repository Participant for Pledge Finance under the Repository Ecosystem for registered/accredited warehouses by WDRA?
    A. Bank of India B. State Bank of India C. Union Bank of India D. Bank of Baroda E. Punjab National Bank

Answer Option: D
Explanation: Bank of Baroda has entered into an agreement in May 2018 with CDSL Commodity Repository Limited (CCRL). Bank of Baroda is the first Public Sector Bank to become the Depository Participant for Pledge Finance under the Repository Ecosystem for registered/accredited warehouses by WDRA.
31. An emerging market economy is highly classified with relatively- one in which the country is becoming a developed nation and is determined through many socio-
    A. Growth factors B. GDP factors C. commercial factors D. external factors E. economic factors

Answer Option: E
Explanation: An emerging market economy is highly classified with relatively - one in which the country is becoming a developed nation and is determined through many socio- economic factors.
32. Which High Court has declared the “entire animal kingdom including avian and aquatic” as legal entities with a distinct persona and corresponding rights, duties and liabilities of a living person?
    A. Kolkata High Court B. Patiala House Courts Complex C. Patna High Court D. Uttarakhand High Court E. Allahabad High Court

Answer Option: D
Explanation: The Uttarakhand High Court on Wednesday declared the “entire animal kingdom including avian and aquatic” as legal entities with a distinct persona and corresponding rights, duties and liabilities of a living person. The Bench comprising Justice Rajiv Sharma and Justice Lok Pal Singh observed, “The Corporations, Hindu idols, holy scriptures, rivers have been declared legal entities and thus, in order to protect and promote greater welfare of animals including avian and aquatic, animals are required to be conferred with the status of legal entity/ legal person. The animals should be healthy, comfortable, well-nourished, safe, able to express innate behavior without pain, fear, and distress. They are entitled to justice.
33. The Reserve Bank of India has asked banks authorized to deal in foreign exchange (Authorized Dealer-I Banks) to share data with-?
    A. Ministry of Finance B. Securities and Exchange Board of India C.Directorate of Revenue Intelligence D. Intelligence Bureau E. Comptroller and Auditor General of India

Answer Option: C
Explanation: The Reserve Bank of India has asked banks authorized to deal in foreign exchange (Authorized DealerI Banks) to share data with the Directorate of Revenue Intelligence (DRI).
34. Bharat Sanchar Nigam Ltd (BSNL) has launched BSNL _____________ a VOIP based service.
    A. TERMS B. WINGS C. NIGAM D. WAVES E. NATER

Answer Option: B
Explanation: Aug 16, 2018- Bharat Sanchar Nigam Ltd (BSNL) has launched BSNL WINGS a VOIP based service. In WINGS, there is no SIM or cable wiring as is a VOIP service through an app.
Q35. Which Airport is the Busiest Airport 2017 by terms of traffic in the world?
    A. Hartsfield–Jackson Atlanta International Airport, USA B.Beijing Capital International Airport, China C.Indira Gandhi International Airport, New Delhi D. Dubai International Airport, UAE E.London Heathrow Airport, UK

Answer Option: A
Explanation: Harts field–Jackson Atlanta International Airport, USA is the Busiest Airport 2017 by terms of traffic in the world
36. Survey points out for the first time in India's history that five States Maharashtra, Gujarat, Karnataka, Tamil Nadu and Telangana account for a whopping ___________________ of India's exports.
    A. 50% B. 80% C. 70% D. 40% E. 60%

Answer Option: C
Explanation: Survey points out for the first time in India's history that five States Maharashtra, Gujarat, Karnataka, Tamil Nadu and Telangana account for a whopping 70 % of India's exports.
37. The government will take steps to boost exports of agriculture commodities which have the potential of reaching _____________ billion.
    A. $100 billion B. $500 billion C. $200 billion D. $700 billion E. $1500 billion

Answer Option: A
Explanation: The government will take steps to boost exports of agriculture commodities which have the potential of reaching $100 billion. The country’s agricultural exports are around $30 billion at present.
38. A dedicated portal of the ECI'S Systematic Voters Education, pursuit of its mission 'leave no voter behind,' with special focus on-
    A. Female B. New Voters C. Senior Citizens D. Persons with Disabilities E. None of the given options is true

Answer Option: D
Explanation: The Election Commission of India (ECI) has organized a two-day “National Consultation on Accessible Elections,” from the 3rd July, 2018 in New Delhi. It inaugurated by the Chief Election Commissioner Shri O.P.Rawat, in presence of the Election Commissioners Shri Sunil Arora and Shri Ashok Lavasa. The event is a part of the ECI’s pursuit of its mission ‘leave no voter behind,’ with special focus on “Persons with Disabilities” (PwD). During the inaugural session, a dedicated portal for the ECI’s ‘Systematic Voters Education and Electoral Participation’ (SVEEP) initiative will also be launched.
39. RBI’s New Rules To Push Another Rs ------------------ Loans Into Insolvency stated Credit Issue.
    A. Rs 5.5 lakh crore B. Rs 4.5 lakh crore C. Rs 3.5 lakh crore D. Rs 2.5 lakh crore E. Rs 1.5 lakh crore

Answer Option: E
Explanation: Another Rs 1.5 trillion (or Rs 1.5 lakh crore) of non-performing assets are now likely to be with the National Company Law Tribunal in the next six months,” according to a Credit Issue report on RBI’s new rules.
40. _____________ is the value of all finished goods and services produced in a country in one year by its nationals.
    A. NPA B. GNP C. CDS D. GDP E. BOP

Answer Option: B
Explanation: Gross national product (GNP) is a broad measure of a nation's total economic activity. GNP is the value of all finished goods and services produced in a country in one year by its nationals.
41. Recently, government set up a ministerial panel, led by ___________, to consider and oversee mergers among the country’s 21 state-run banks.
    A. Urjit Patel B. Arun Jaitley C. Hasmukh Adhia D. Rajiv Kumar E. Dr. Viral V. Acharya

Answer Option: B
Explanation: The government set up a ministerial panel, led by finance minister Arun Jaitley, to consider and oversee mergers among the country’s 21 state-run banks.
42. Who among the following is the author of e-book ‘India 2017 Year Book’?
    A. Rajiv Gauba B. Rajiv Mehrishi C. R.K Verma D. A.K. Lohani E. None of these

Answer Option: B
Explanation: Rajiv Mehrishi, the Comptroller and Auditor General of India (CAG) has authored an e-book ‘India 2017 Yea Book’ on current affairs in India. It is recently launched in Rajasthan.
43. Which of the following countries has been selected as the host of Commonwealth Games 2018
    A. India B. Australia C. Pakistan D. South Africa E. Sri Lanka

Answer Option: B
Explanation: Commonwealth Games 2018 will be held in Gold Coast, Australia from April 4 to April 15 2018. Hence, option B is correct.
44. Under the Union Budget 2017, provision under MGNREGA (Mahatma Gandhi National Rural Employment Guarantee Act) has been increased to _____?
    A.Rs. 38500 crores B.Rs. 40000 crores C.Rs. 45000 crores D.Rs. 48000 crores E. None of the above

Answer Option: D
Explanation: Under the Union Budget 2017, provision under MGNREGA (Mahatma Gandhi National Rural Employment Guarantee Act) has been increased to Rs. 48000 crores in 2017-18 against Rs. 38,500 crores in 2016-17, i.e. the highest ever.
45. Headquarters of Amnesty International is at _______.
    A. New York B. London C. Washington D. Berlin E. None of these

Answer Option: B
Explanation: Amnesty International is a non-governmental organization focused on human rights with over 7 million members and supporters around the world. Headquarter of Amnesty International is at London, United Kingdom.
46. Which of the following country has won the ‘2017 FIFA Under-17 World Cup’?
    A. India B. Germany C. England D. Spain E. None of these

Answer Option: C
Explanation: England has won the ‘2017 FIFA Under-17 World Cup’ by defeating European Champions Spain at Salt Lake stadium in Kolkata. England’s goal-machine Rhian Brewster won the golden boot.
47. In the financial term “SWIFT”, what is ‘S’ stand for?
    A. Society B. Saving C. Scheme D. Securitization E. Standard

Answer Option: A
Explanation: The Society for Worldwide Inter bank Financial Telecommunication (SWIFT) provides a network that enables financial institutions worldwide to send and receive information about financial transactions in a secure, standardized and reliable environment.
48. There were six agreements signed between India and Italy during the two days visit of Italy PM. Who is the current prime minister of Italy?
    A. Eduardo Philippe B. Paolo Gentiloni C. Emmanuel Macron D. Angela Merkel E. Mariano Rajoy Brey

Answer Option: B
Explanation: There were six agreements signed between India and Italy during the two days visit of Italy PM. Paolo Gentiloni is the current prime minister of Italy.
49. Ease of Doing Business Index is released by
    A.World Bank B.World Economic Forum (WEF) C.International Monetary Fund (IMF) D.World Trade Organization (WTO) E.United Nations Development Program (UNDP)

Answer Option: A
Explanation: It is released as part of the World Bank’s annual report titled Doing Business 2017: Equal Opportunity for All. It was introduced in 2004. The ranking of country is based on index averages the country’s percentile rankings on 10 indicators each having equal weight age.
50. The World Economic outlook, a survey conducted and published by
    A. World Economic Forum (WEF) B. World Bank (WB) C. United Nations Development Program (UNDP) D. World Trade Organization (WTO) E. International Monetary Fund (IMF)

Answer Option: E
Explanation: The World Economic outlook, a survey conducted and published by International Monetary Fund (IMF). It is published biannually and partly updated two times a year.
Directions(1 - 5): Find out the wrong no. in the following series?
1. 59, 60, 61,64 ,79 ,185 ,1129
    A. 185 B. 61 C. 60 D. 1129 E. 79

Answer Option: A
Explanation: The differences of the series is 1, 3, 15, 106 ….. This can be formed on the basis like 1 × 1,1 × 3,3 × 5,15 × 7..so on Hence the wrong no. is 185.
2. 330, 80, 280, 120, 250, 130, 240
    A. 330 B. 130 C. 240 D. 120 E. 250

Answer Option: B
Explanation: The difference of 330 and 80 = 250 The difference of 80 and 280 = 200 The difference of 280 and 120 = 160 The difference of 120 and 250 = 130 The difference of 250 and 130 = 120 And this difference will be 110, so the wrong no. is 130
3. 291, 147, 75, 39, 22,12,7.5
    A. 39 B. 147 C. 22 D. 7.5 E. 291

Answer Option: C
Explanation: The series is 291 − 147 = 144 147 − 75 = 72 75 − 39 = 36 39 − 22 = 17 but this difference will be 18 So the wrong no. is 22
4. 1, 3, 9, 31, 129, 652
    A. 1 B. 3 C. 652 D. 31 E. 129

Answer Option: C
Explanation: The series is 1 × 1 + 2 = 3 3 × 2 + 3 = 9 9 × 3 + 4 = 31 31 × 4 + 5 = 129 129 × 5 + 6 = 651 Hence the no. 652 is wrong
5. 26, 27, 34, 58, 106, 186, 306
    A. 27 B. 34 C. 186 D. 106 E. 306

Answer Option: A
Explanation: the series is 26 + 12 − 1 = 26 26 + 32 − 1 =34 34 + 52 − 1 = 58 58 + 72 − 1 = 106 and so on…..
Directions (6 - 10): Bar graph given below shows pens sold by a retailor on five different days. Study the data carefully and answer the following questions. Pen sold on different days

6. Out of total pens sold on Tuesday ratio between total defective pens sold to total pens sold is 7: 15. Find total number of non - defective pens sold on Tuesday by retailer?
    A. 40 B. 15 C. 60 D. 45 E. 90

Answer Option: A
Explanation: Non defective pens sold on Thursday= 75/15×8=40
7.Total number of pens sold on Saturday is 40% more than total number of pens sold on Wednesday. Find total number of pens sold on Friday and Saturday together?
    A. 92 B. 122 C. 172 D. 125 E. 105

Answer Option: A
Explanation: pens sold on Saturday = 30 × 1.4 = 42 Pens sold on Friday and Saturday = 50 + 42 = 92
8.Find the difference between total number of pens sold on Monday and Tuesday together to total number of pens sold on Thursday and Friday together?
    A. 25 B. 40 C. 5 D. 22 E. 10

Answer Option: C
Explanation: Required difference = 25 + 75 − 50 = 5
9.Total number of pens sold on Tuesday is 25% more than total number of pens sold on Sunday. Find total of pens sold on Sunday?
    A. 72 B. 60 C. 94 D. 43 E. 75

Answer Option: B
Explanation: Pens sold on Sunday= [latex]\frac{75}{125}[/latex] × 100 = 60
10. Out of the total pens sold on Thursday,20% are blue ink pen. Out of remaining 25% are red ink pen and remaining are black in pen. Find total number of blue and black in pen. Find total no. of blue and black ink pen sold on Thursday?
    A. 36 B. 46 C. 56 D. 66 E. 55

Answer Option: A
Explanation: Sale of blue ink pen on Thursday = 45 × [latex]\frac{20}{100}[/latex] = 9 Sale of Red ink pen on Thursday = (45−9) × [latex]\frac{25}{100}[/latex] = 9 Sale of Black ink pen on Thursday = (45−9) × [latex]\frac{75}{100}[/latex] = 27 Required sum = 9 + 27 = 36
Direction (11 - 15): Pie chart given below shows total number of workers in three different companies. Table given below shows ratio between officers and workers working in these companies. Study the data carefully and answer the following questions

company officers : workers
A 1 : 16
B 1 : 18
C 1 : 12

11. Find the difference between total number of workers in company ‘A’ and total number of workers in company ‘B’ and ‘C’ together?
    A. 324 B. 423 C. 523 D. 624 E. 124

Answer Option: A
Explanation: Required difference= [latex]\frac{900}{100}[/latex] × (44 + 24 − 32) = 324
12. Total number of employees in company ‘B’ is how much more than total number of employees in company ‘C’.
    A. 184 B. 176 C. 225 D. 265 E. 125

Answer Option: A
Explanation: Required difference = 900 × [latex]\frac{44}{100}[/latex] × [latex]\frac{19}{18}[/latex] − 900 × [latex]\frac{24}{100}[/latex] × [latex]\frac{13}{12}[/latex] = 184
13. Total number of officers in company ‘A’ is how much less than total number of officers in company ‘B’?
    A. 1 B. 7 C. 9 D. 4 E. 8

Answer Option: D
Explanation: Required difference = 900 × [latex]\frac{32}{100}[/latex] × [latex]\frac{9}{16}[/latex] − 900 × [latex]\frac{1}{18}[/latex] × [latex]\frac{44}{100}[/latex] = 4
14. Find the ratio between total number of workers in company A and C together to total number of officers in company A and C together?
    A. 17: 1 B. 12: 7 C. 14: 1 D. 18: 1 E. 22: 3

Answer Option: C
Explanation: Required ratio = 504 : 36 = 14 : 1
15. Total number of officers and workers in company D is 50% and 25% more than total number of officers and workers in company ‘C’ respectively. Find total number of employees in company ‘D’?
    A. 297 B. 279 C. 972 D. 927 E. 227

Answer Option: A
Explanation: Total no. of employees in company D = 900 × [latex]\frac{24}{100}[/latex] × [latex]\frac{1}{12}[/latex] × 1.25 −900 × [latex]\frac{24}{100}[/latex] × 1.5 = 297
Direction (16 - 20): Choose correct option:
    A. Quantity I > Quantity II B. Quantity I < Quantity II C. Quantity I ≥ Quantity II D. Quantity I ≤ Quantity II E. Quantity I = Quantity II or No relation

16. If a speed of boat is 500% more than the speed of a current. Quantity I – ‘X’: If boat can travel a distance of 63 km 3 hr, in downstream then ‘X’ is the speed of the boat in upstream (km/hr.) Quantity II- 15 km/hr
Answer Option: E
Explanation: Quantity I: Let speed of current = x Speed of boat = x + 5x Downstream speed = 7x [latex]\frac{63}{7x}[/latex] = 3 X = 3 Upstream speed = 6x – x = 5x = 15kmph Quantity II- 15 km/hr Hence QuantityI = Quantity II or No relation
17. Sum of 8 Consecutive even number is S1. Quantity I – Sum of second number and eight number in S1 Quantity II – Sum of the third number and sixth number in S1
Answer Option: A
Explanation: Let no. be x, x+2, x+4, x+6 ….. Quantity I = x + 2 + x + 14 = 2x + 16 Quantity II= x + 4 + x + 10 = 2x + 14 Hence Quantity I > Quantity II
18. Quantity I. ‘X’: X[latex]^{2}[/latex] + X – 6 = 0 Quantity II. ‘y’: y[latex]^{2}[/latex] + 7y + 12 = 0
Answer Option: A
Explanation: Qunatity I = x = −3, 2 Qunatity II = y = −4, −3 Quantity I > Quantity II
19. An article is sold at Rs. 1500 after allowing discount of 12.5% on Marked price. Quantity I – Rs. 550 Quantity II – the Marked price of the article
Answer Option: A
Explanation: Quantity I – Rs. 550 Quantity II – let Marked price of article = M M × [latex]\frac{7}{8}[/latex] = 1500 M = [latex]\frac{1200}{7}[/latex] Hence Quantity I > Quantity II
20. A’s efficiency is 25% more than B Quantity I – ‘X’: A can-do [latex]\frac{5}{6}[/latex]th of total work in ‘X’ days Quantity II – ‘Y’: B can do [latex]\frac{4}{5}[/latex]th of total work in ‘Y’ days
Answer Option: A
Explanation: Quantity I = 10 days Quantity II = 12 days Hence Qunatity1 Qunatity11
21. Sum of volume of cylinder (S) and volume off cone (C) is 2190π cm2& height of both cylinder and cone is same i.e, 10 cm. if radius of cone is 15 cm find the ratio of radius of S to radius of C?
    A. 4 : 5 B. 3 : 7 C. 3 : 5 D. 7 : 5 E. 1 : 5

Answer Option: A
Explanation: Let r , h be the dimensions of cylinder And R , H be the dimensions of the cone ATQ h = H = 10 πr[latex]^{2}[/latex] + 1/3πR[latex]^{2}[/latex]h = 2190π r = 12 R= 15 So r:R = 4 : 5
Direction(22 - 26): Find the value of (?) in following approximation questions:
22. 2? = 32. 01 + 128.01 × 1023.99 + 7.99
    A. 5 B. 1 C. 2 D. 3 E. 4

Answer Option: A
Explanation: ?=5
23. (? + 9.97) × 12.8 = 20.12% of 1319.97
    A. 210 B. 220 C. 270 D. 290 E. 205

Answer Option: B
Explanation: ?=220
24. (39.99% of [latex]\frac{2}{5}[/latex] + 60.05% of [latex]\frac{1}{5}[/latex]) × 103 + ? = 1
    A.369 B.– 369 C.279 D.– 279 E.– 159

Answer Option: D
Explanation: (39.99% of [latex]\frac{2}{5}[/latex] + 60.05% of [latex]\frac{1}{5}[/latex]) × 103 + ? = 1 ( 40% of [latex]\frac{2}{5}[/latex] + 60% of [latex]\frac{1}{5}[/latex]) × 1000 + ? ≈ 1 ? = 1 – (0.16 + 0.12) × 1000 ? = 1 – 0.28 × 1000 = 1 – 280 = – 279
25. 12.992 + ?[latex]^{2}[/latex] = 11.102 + 14.0092
    A. 29929 B. 383 C. 146689 D. 148 E. 293

Answer Option: D
Explanation: 12.992 + ?[latex]^{2}[/latex] = 11.102 + 14.0092 132 + ? ≈ 112 + 142 ? = (121 + 196) – 169 ? = 148
26. (22.99 + 17.01) ÷ 1.998 × 3.997 – 41.998 + 644.199 = ?
    A. 798 B. 542 C. 682 D. 745 E. 762

Answer Option: C
Explanation: (22.99 + 17.01) ÷ 1.998 × 3.997 – 41.998 + 644.199 = ? [latex]\frac{23 + 17}{2}[/latex] × 4 – 42 + 644 ≈ ? ? = 20 × 4 – 42 + 644 ? = 80 + 644 – 42 = 724 – 42 = 682 There are three persons A, B and C who each invested in two different scheme S1 and S2. A in invested Rs. 80,000 for 2 yr in scheme S1 and 30,000 for 4 years in scheme S2. B invested Rs 30,000 for year in S1 and he did not invest in scheme B. B also obtained a profit of 10,000 by selling his car. C invested Rs 50000 for 5 years in scheme S1 and 10000 for 3 year in scheme S2. Total profit obtained from scheme S1 is 2 lakh and scheme S2 is 90,000.
27. Profit obtained by A from scheme S1 is what percent of profit obtained by C from scheme S2.
    A. 326 8/9% B. 317 8/9% C. 326 5/9% D. 335 5/9% E. 355 5/9%

Answer Option: E
Explanation: Ratio of profit of A, B and C in scheme S1 80000 × 2 : 30000 × 3 : 50000 × 5 16 : 9 : 25 In scheme S1 Profit of A = 64000 Profit of B = 36000 Profit of C = 100000 Ratio of profit of A and C in scheme S2 12 : 3 Profit of A in scheme S2 = 72000 Profit of B in scheme S2 = 18000 64000 × [latex]\frac{100}{1800}[/latex] = 335 %
28. If A had invested his sum at Simple interest for 3 yr at the rate of R% p.a. instead in scheme S1 and B has invested his sum at compound interest at (R + 5%) p.a. for 1 year and difference in interest obtained is 30,000 then find value of R%
    A. 15% B. 7% C. 5% D. 8% E. 2%

Answer Option: A
Explanation: [latex]\frac{(80000 × R × 3)}{100}[/latex]-[3000 × {[latex]\frac{(R+5)}{100}[/latex]}]=30000 R = 15%
29. If sum of investment of A in both schemes and total profit obtained by A from both scheme is invested at compound interest at the rate of 20% p.a. then find the total compound interest obtained in 2 yr
    A. 106240 B. 108240 C. 102208 D. 123480 E. 106500

Answer Option: B
Explanation: Total investment of A = 110000 Total profit of A = 136000 Rate of interest = 44% CI=[latex]\frac{44}{100}[/latex] × (136000 + 110000) = 108240
30. What is the average of profit attained by A from scheme S1 and profit of C obtained from scheme S2.
    A. 43000 B. 41000 C. 54000 D. 56000 E. 47000

Answer Option: B
Explanation: Required average = 41000
31. What is the ratio of total profit obtained by B and profit obtained by C from scheme S1
    A. 23: 47 B. 54: 47 C. 36: 43 D. 23: 50 E. 27: 50

Answer Option: D
Explanation: Required ratio = (36000 + 10000) : 100000 = 23 : 50
32. What is the probability of forming word from the letters of word “IMPEACH” such that all vowels come together?
    A. [latex]\frac{1}{7}[/latex] B. [latex]\frac{4}{7}[/latex] C. [latex]\frac{3}{7}[/latex] D. [latex]\frac{4}{35}[/latex] E. [latex]\frac{13}{35}[/latex]

Answer Option: A
Explanation: Total numbers of ways → 7! Favorable numbers of ways → 5! ×3! Probability → = [latex]\frac{1}{7}[/latex]
33.Sum of A’s and B’s age 6 years ago is 88. A’s age 18 yrs ago is equal to B’s age 6 years ago. Find the age of A two year hence?
    A. 51 yrs B. 62 yrs C. 22 yrs D. 58 yr E. 18 yrs

Answer Option: D
Explanation: Let present age of A be x yrs& present age of B be y yrs. ATQ X + y = 88 + 12 X + y = 100 …… (!) X – 18 = y - 6 X – y = 12 ……. (!!) Solving (i) & (ii) X = 56 Age of A 2 year here = 58 yrs
34.In a box there are 6 blue ball, X red balls & 10 green balls. Probability of choosing one red ball from the given box is [latex]\frac{1}{3}[/latex], then find the sum of red and blue balls in the box?
    A. 14 B. 11 C. 10 D. 17 E. 15

Answer Option: A
Explanation: ATQ = X = 8 Sum of red and blue ball = 8 + 6 = 14
35. Train A of length 120 m can cross a platform of length 240m is in 18 second the ratio of speed train A and Train B is 4: 5. The find the length of train B if train B can cross a pole in 12 seconds.
    A. 300m B. 330m C. 310m D. 370m E. 340m

Answer Option: A
Explanation: Let speed of train A be S S × 18 = 360 S = 20 m/s A: B = 4 : 5 A: B = 4 : 5 Speed of B = 25m/s Length of train B = 25 × 12 = 300m
Direction (36 - 40): Study the information given below and answer the questions based on it. The bar chart shows the discount percentage allowed on six different articles on two different days of the week i.e. Monday and Tuesday. The table provides us with the marked price of different articles. Some of the data are missing for the table.
Note:The cost price and the market price of the articles on both days are the same.
Article Marked Price
A ---
B 1500
C 1400
D ----
E 1200
F ---

36. If the difference in selling price of the article F on both the days is Rs. 100, then find the cost price of the same article if the profit obtained on Monday through F is 25%?
    A. 500 B. 600 C. 700 D. 800 E. 750

Answer Option: B
Explanation: Let the marked price of the article F on both days = 100 Discount on Monday = 25% So selling price on Monday = 75 Discount on Tuesday = 35% So selling price on Tuesday = 65 Difference = (75 - 65) = 10 Then it is given difference of selling price=100 10% = 100 1% = 10 So selling price on Monday, 75 × 10 = 750 As profit is given on Monday then, 125% of CP of Monday = 750 CP = 600rs.
37. If the profit on article D on Tuesday and that of article A on Tuesday is 20% and 27.5% respectively. Then find the ratio of cost price of article D and article A, if marked price of A is [latex]\frac{3}{4}^{th}[/latex] of the marked price of article D?
    A. 5:6 B. 4:5 C. 5:4 D. 3:5 E. 5:8

Answer Option: C
Explanation: Let the marked price of article D = 400x Let the marked price of article A = 300x Discount of article D on Tuesday = 25% Selling price of article D = 400x × [latex]\frac{75}{100}[/latex] = 300x Discount of article A on Tuesday = 15% Selling price of article D = 300x × [latex]\frac{85}{100}[/latex] = 255x Profit of article D = 20% Profit of article A = 27.5% Ratio of cost prices of both articles, CPD × [latex]\frac{120}{CPA}[/latex]% × 127.5% = [latex]\frac{SPD}{SPA}[/latex] CPD × [latex]\frac{120}{CPA}[/latex]% × 127.5% = [latex]\frac{300x}{255x}[/latex] [latex]\frac{CPD}{CPA}[/latex] = 300 × [latex]\frac{1275}{255}[/latex] × 1200 [latex]\frac{CPD}{CPA}[/latex] = [latex]\frac{5}{4}[/latex]
38. Total selling price of article A on Tuesday and article E on Monday is 1590. Then find the profit of article C on Monday, if the cost price of article C is [latex]\frac{5}{4}^{th}[/latex] of the marked price of article A?
    A. 110 B. 80 C. 250 D. 90 E. 125

Answer Option: D
Explanation: Marked price of article E = 1200 Discount on Monday of article E = 10% So selling price of article E on Monday = 1200 × [latex]\frac{90}{7=100}[/latex] = 1080 Total selling price of article A on Tuesday and article E on Monday is 1590 So selling price of article A on Tuesday = 1590 - 1080 = 510 Discount on Tuesday for article A = 15% Let the marked price of article A = 100 So selling price of article A on Tuesday = 100 - 15 = 85% Now, 85% = 510 Then 100% = 600 So marked price of article A = 600 Cost price of article C is [latex]\frac{5}{4}[/latex] of the marked price of article A, So cost price of article C = 600 × [latex]\frac{4}{5}[/latex] = 750 Marked price of article C = 1400 Discount on Monday = 40% So selling price of article C on Monday = 1400 × [latex]\frac{60}{100}[/latex] = 840 So profit of article C on Monday = 840 - 750 = 90
39. What is the sum of the selling prices of article B on Monday, article C and article E on Tuesday
    A. 3100 B. 2165 C. 3085 D. 2750 E. 1790

Answer Option: C
Explanation: Marked price of article B = 1500 Discount on Monday = 25% Selling price of article B on Monday = 1500 × [latex]\frac{75}{100}[/latex] = 1125 Marked price of article C=1400 Discount on Tuesday = 20% Selling price of article B on Monday = 1400 × [latex]\frac{80}{100}[/latex] = 1120 Marked price of article E = 1200 Discount on Tuesday = 30% Selling price of article B on Monday = 1200 × [latex]\frac{70}{100}[/latex] = 840 Total selling price=1125 + 1120 + 840 = 3085
40. If the total cost price of article B and article D is 1500. Then find the cost price of article D, if the profit of article B on Tuesday is 5%?
    A. 700 B. 600 C. 800 D. 550 E. 500

Answer Option: E
Explanation: Marked price of article B = 1500 Discount of article B on Tuesday = 30% Selling price of article B on Tuesday = 1500 × [latex]\frac{70}{100}[/latex] = 1050 Profit of article B on Tuesday = 5% So, 105% = 1050 Then CP of article B = 1050 × [latex]\frac{100}{105}[/latex] = 1000 Total CP of article B and article D = 1500 CP of article D = 1500 - 1000 = 500
41. B is 200% efficient than A and 50% efficient than C, If all of them work together for 2 days to complete the work, then in how many days A alone can complete the work?
    A.10 days B.12 Days C.4 days D.6 days E.8 days

Answer Option: B
Explanation: Let A’s work=100 B’s work=100 × [latex]\frac{100}{300}[/latex] = 300 C’s work=300 × [latex]\frac{100}{150}[/latex] = 200 A : B : C = 100 : 300 : 200 = 1: 3 :2 Total work 6x × 2 = 12x So A’s work= [latex]\frac{12}{1}[/latex] = 12 days B’s work = [latex]\frac{12}{3}[/latex] = 4 days C’s work= [latex]\frac{12}{2}[/latex] = 6 days So A alone can complete the work in 12 days. Hence, option B.
42. The sum of ages of A, B and C is 81 years. One year before the ratio of ages of A&B was 1:2. One year before the ratio of ages of B&C was 4:7. So find the age of B after 6 years?
    A. 33 years B. 35 years C. 42 years D. 31 years E. 29 years

Answer Option: D
Explanation: One year before age, A : B = 1 : 2 B : C = 4 : 7 So final ratio, A : B : C = 4 : 8 : 14 = 2 : 4 :7 Present ratio, 2x + 1 : 4x + 1 :7x + 1 According to the question, 2x + 1+ 4x + 1 + 7x + 1 = 81 13x + 3 = 81 X = 6 So present age of B, 4 × 6 + 1 = 25 After 6 years, 25 + 6 = 31 years.
43. If the sum of circumference of a circle and the circumference of a semicircle of same radius (R) is 400cm. Then find the radius of another circle whose radius is 200% more than R?
    A. 75cm B. 105cm C. 63cm D. 48cm E. 42cm

Answer Option: B
Explanation: Circumference of a circle = 2πR Circumference of a circle = πR + 2R So, 2πR + πR + 2R = 400 3πR+2R = 400 R(3π+2) = 400 R (3 × [latex]\frac{22}{7}[/latex] + 2) = 400 R × [latex]\frac{80}{7}[/latex] = 400 R = 400 × [latex]\frac{7}{80}[/latex] R = 35 cm Radius of another circle whose radius is 200% more than R, So another circle radius will be R + 2R = 3R 3 × 35 = 105cm
44. There are two vessels A&B having quantities milk and water. The ratio of quantity in vessel is 6:7 and both of them having equal quantity of water. The quantity of pure milk in vessel A and B respectively 40L and 50L. Now these two vessels A and B quantities put in another vessel C, Find the ratio of milk and water in vessel C?
    A. 9 : 4 B. 3 : 4 C. 5 : 6 D. 7 : 5 E. 8 : 11

Answer Option: A
Explanation: Let the quantity of water = x According to the question, 40 + [latex]\frac{x}{50}[/latex] + x = [latex]\frac{6}{7}[/latex] 280 + 7x = 300 + 6x X = 20 So quantity of water in each vessel = 20L After putting into vessel C, Total milk = 40 + 50 = 90L Total wate r= 20 + 20 = 40L So milk/water = [latex]\frac{90}{40}[/latex] = 9 : 4
45. A shopkeeper buys a watch and a speaker for a total of Rs.1600. He sells the watch at 30% profit and the speaker at a profit of 20% after offering a discount of 20% on the marked price for speaker. If his total profit is 25.625%, how much more profit he would have obtained, had he sold the speaker at its marked price
    A. Rs.225 B. Rs.180 C. Rs.210 D. Rs.216 E. None of these

Answer Option: C
Explanation: Let the cost price of the watch be x. Hence the cost price of the speaker = 1600 - x. Profit obtained = Profit % × [latex]\frac{Cost Price}{100}[/latex] Hence Profit obtained on the watch = 30 × [latex]\frac{x}{100}[/latex] = 0.3x. Profit obtained on the speaker = 20 × [latex]\frac{1600 - x}{100}[/latex] = 320 - 0.2x. Total profit = 320 + 0.3x-0.2x = 320 + 0.1x. But profit = 25.625% = 25.625 × [latex]\frac{1600}{100}[/latex] = Rs.410. Hence, 320+ 0.1x = 410, 0.1x = 410 - 320 = 90. Hence x = [latex]\frac{90}{0.1}[/latex] = Rs.900. Hence, the cost price of the speaker = 1600 - 900 = Rs.700. Selling price = Cost Price + Profit = 700 + 700 × [latex]\frac{20}{100}[/latex] = 700 + 140 = Rs. 840. Let the marked price be y. Hence 840 = y - 20 × [latex]\frac{y}{100}[/latex] = 0.8y. (Since a discount of 20% was offered) Hence y = [latex]\frac{840}{0.8}[/latex] = Rs.1050. Difference between market price and selling price = 1050 - 840 = Rs.210. Hence the shopkeeper would have obtained Rs. 210 more if he would have sold the speaker at its marked price.
Direction (46-50): Study the information given below and answer the questions based on it. There are three highways i.e. Highways A, Highways B and Highways C, Some numbers of vehicles on these highways on different days i.e. Friday, Saturday and Sunday. On each day some numbers of vehicles pass on these three highways.
Friday: The number of vehicles on highway B is one-third of highway B of Saturday. The number of vehicles on highway C is 3/4th of highway C of Saturday. The number of vehicle on highway A and C is the same.
Saturday: Total number of vehicle of Saturday is 75000. The number of vehicles on highway B is 15000 less than that of highway A & C together. The number of vehicles on highway A is 5/6th of highway A of Sunday.
Sunday: Total number of vehicles of Sunday is 4/5th of Saturday. The number of vehicle of highway A is equal to highway B of Saturday. The number of vehicles on highway C is the same on Saturday and Sunday.
46. What is total number of vehicles on Friday?
    A. 45000 B. 40000 C. 50000 D. 55000 E. 48000

Answer Option: B
Explanation:
47. What is the total number of vehicles on Highway A of three days?
    A. 75000 B. 65000 C. 60000 D. 70000 E. 80000

Answer Option: D
Explanation: The total number of vehicles on highway A of three days, 15000+25000+30000=70000
48.What is the average number of vehicles on Sunday?
    A. 15000 B. 20000 C. 25000 D. 10000 E. 12000

Answer Option: B
Explanation:
Day High Way A High Way B High Way C
Friday 15000 100000 15000
Saturday 25000 30000 20000
Sunday 30000 10000 20000

49. What is difference between the vehicles on highway A of Saturday and highway C of Friday?
    A. 5000 B. 15000 C. 10000 D. 20000 E. 8000

Answer Option: C
Explanation:
50. The number of vehicles on highway B on Friday is approximately what percent of total vehicles on Sunday?
    A. 15% B. 18% C. 12% D. 17% E. 21%

Answer Option: D
Direction (1-5): Study the following information carefully to answer the given questions.
A word and number arrangement machine when given an input line of words and numbers rearranges them following a particular rule. The following is an illustration of input and rearrangement.
Input : 30 69 20 96 50 56 76 19 Step I : 20 30 69 96 50 56 76 21 Step II : 31 20 69 96 56 76 21 51 Step III : 57 31 20 96 76 21 51 70 Step IV : 77 57 31 20 21 51 70 97 and Step IV is the last step of the rearrangement the above input.
As per the rules followed in the above steps, find out in each of the following questions the appropriate steps for the given input given below: Input: 50 17 85 34 88 55 11 92 10 78
1. How many steps will be needed to complete the arrangement?
    A. Five B. Six C. Four D. Seven E. None of these

Answer Option: A
Explanation: In the arrangement, number are arranged as least number is arranged from left end and second least number is arranged from right after adding +1 in both sides. Same rule followed till the arrangement is completed. Input: 50 17 85 34 88 55 11 92 10 78 Step I: 11 50 17 85 34 88 55 92 78 12 Step II: 18 11 50 85 88 55 92 78 12 35 Step III: 51 18 11 85 88 92 78 12 35 56 Step IV: 79 51 18 11 88 92 12 35 56 86 Step V: 89 79 51 18 11 12 35 56 86 93 Five steps will be needed to complete the arrangement.
2. Which of the following would be Step IV?
    A. 79 50 18 11 88 92 12 35 56 86 B. 79 51 18 10 88 92 12 35 56 86 C. 79 51 18 11 88 92 12 35 56 86 D. 79 51 18 11 87 92 12 35 56 86 E. None of these

Answer Option: C
3. In Step V, which of the following number would be at 6th position from the right?
    A. 88 B. 92 C. 11 D. 12 E. 17

Answer Option: D
4. Which of the following would be the final arrangement?
    A. 89 79 51 18 11 12 35 55 86 93 B. 89 79 51 18 11 12 35 56 86 93 C. 89 79 51 18 11 12 34 56 86 93 D. 89 79 51 18 10 12 35 56 86 93 E. None of these

Answer Option: B
5. What will be the position of '92' in step IV?
    A. Fifth from the left B. Eight from the left C. Fifth from the right D. Sixth from the right E. None of these

Answer Option: C
Direction (6-10): Study the following information carefully to answer the questions. In a certain code language, 'business is easy to do' is written as 'pa ro se yo ze', 'business arise difficulty in life' is written as 'lo fe pa so te', 'life is easy' is written as 'so ro yo', 'difficulty to do work' is written as 'se lo ze me' and 'easy business life' is written as 'pa yo so'
6.What may be the possible code for 'difficulty arise in business'?
    A.lo, te, fe, pa B.lo, te, so, pa C.. lo, so, pa, yo D.te, lo, me, so E.. lo, fe, pa, ro

Answer Option: A
Explanation: As per given information coding description is as follows: 'business is easy to do' - 'pa ro se yo ze'……1) 'business aries difficulty in life' - 'lo fe pa so te'. ……2) 'life is easy' - 'so ro yo'. ……3) 'difficulty to do work' - 'se lo ze me' ……4) 'easy business life' - 'pa yo so'. ……5) From 1) and 2) business is coded as: pa From 1), 2) and 5) easy is coded as: yo From 1), 2) and 5) life is coded as: so From 1) and 3) ‘is’ is coded as: ro From 2) and 4) ‘difficulty’ is coded as: lo From 1), 2), 3) and 4) ‘to’ is coded as: Either ‘se’ or ‘ze’ From 1), 2), 3) and 4) ‘do’ is coded as Either ‘se’ or ‘ze’ From 1), 2), 3) and 4) ‘work’ is coded as me From 1), 2), 3) and 4) ‘aries’ is coded as Either ‘fe’ or ‘te’ From 1), 2), 3) and 4) ‘in’ is coded as Either ‘fe’ or ‘te’ ‘lo, te, fe, pa’ is the code for 'difficulty aries in business'.
7. What may be the possible code for 'life is complicated' in the given code language?
    A.so ro fe B.so pa ro C.ze pa ro D.so fa ro E.None of these

Answer Option: D
8.What is the code for 'arise' in the given code language?
    A. fe B. te C. lo D. pa E. Either 'fe' or 'te'

Answer Option: E
9.What is the code for 'difficulty' in the given code language?
    A. pa B. lo C. so D. te E. yo

Answer Option: B
10. What is the code for 'do' in the given code language?
    A. se B. ze C. ro D. pa E. Either 'se' or 'ze

Answer Option: E
Explanation: Direction (11-12): Study the given question carefully and answer the given question.
C is the daughter of the one who is the son of B. There is only one married couple in the group. F is the brother of E, who is the daughter of D and granddaughter of B. A is the mother of C, who is a married couple in the group.
11. What is true with respect to F?
    A. F is the son of B and A B. F is the daughter of C and E C. F is the son of D and A D. F is daughter of B E. None of these

Answer Option: C
Explanation: As per given information family tree is as follows:
null
12. How is B related to E?
    A. B is uncle of E B. B is grandfather of E C. B is Grandmother of E D. B is brother of E E. Can’t be determined

Answer Option: E
Direction (13-17): Study the following information to answer the given questions. Fourteen people are sitting in two parallel rows containing seven people each, in such a way that there is an equal distance between adjacent persons.
In row 1, P, Q, R, S, T, U, and V are seated and all of them are facing south. In row 2, A, B, C, D, E, F, and G are seated and all of them are facing north. Therefore, in the given seating arrangement each member seated in a row faces another member of the other row. U sits opposite to G and are in the middle of the rows. A sits fourth to the right of D. Neither A nor D sits at extreme ends. T faces D. V does not face A and V does not sit at any of the extreme ends. V is not an immediate neighbor of T. B sits at one of the extreme ends. Only three people sit between B and E. E does not face V. Three persons sit between R and Q. R is not an immediate neighbor of T. C does not face V. P is not an immediate neighbor of R
13. Who amongst the following sit at extreme left ends of the parallel rows?
    A. R, B B. F, T C. P, C D. B, F E. None of these

Answer Option: C
14. Who amongst the following faces A?
    A. R B. T C. P D. Q E. S

Answer Option: E
15. How many persons are seated between T and S?
    A.One B.Two C.Three D.Four E.None of these

Answer Option: C
16. P is related to V in the same way as C is related to F. Which of the following is E related to, following the same pattern?
    A. B B. D C. C D. A E. None of the above

Answer Option: A
17. Which of the following is true regarding F?
    A. F sits second to right of C. B. F is not an immediate neighbor of A C. F sits third to left of D D. F sits at one of the extreme ends of the line E. F faces V

Answer Option: E
Direction (18-22): Study the given information carefully and answer the following question.
A, B, C, D, E, F, and G are seven different boxes packed with different numbers written on top of them viz. 1, 2, 3, 4, 5, 6 and 7 but not necessarily in the same order. Box D is immediately above the box packed with number 5. More than three boxes are there between box packed with number 4 and box packed with number 2. There are two boxes between the box B and box E. There is only one box between box E and box G. There are three boxes between box G and box A, which is packed with number 1. There are only two boxes between box A and box C, which is, packed with number 6. The box packed with number 2 is immediately above the box G. More than three boxes are there between box packed with number 5 and box packed with number 4. Box E is packed with number 3
18. Which of the following box is packed with a number which is a perfect square?
    A. D B. B C. F D. G E.Cannot be determined

Answer Option: B
Explanation:
Box Number
D 2
G 5
C 6
E 3
F 7
A 1
B 4

19. How many boxes are there between box A and box which is packed with number 5?
    A. None B. One C. Two D. Three E. Four

Answer Option: D
20. What is the sum of numbers of box D and box F?
    A. Five B. Six C. Seven D. Eight E. Nine

Answer Option: E
21. Which of the following box is packed with even number?
    A. A B. G C. D D. E E. F

Answer Option: C
22. Which box is packed with number 7?
    A. D B. F C. B D. G E. C

Answer Option: B
Direction (23-27): Study the given information carefully and answer the following question.
There are eight friends in a group M, N, O, P, Q, R, S, and T. They have their birthdays in one of the following months – March, June, November, and December. In each month there are two birthdays. The birthdays are either on 16th or 25th of any given month. O and Q have birthdays in a month having 31 days. There are at least two birthdays between M and T. M does not have a birthday in March. N has a birthday in November. T’s birthday is immediately after N’s. Q’s birthday is not on 16th of any month. N’s birthday is somewhere after O’s. There are two birthdays between O and P. N and Q have a birthday on the same date. Similarly, M and S have birthdays on the same date. P and T do not have birthdays on the same date. R does not like December month
23. Who among the following has birthday on 25th of June?
    A. O B. P C. R D. S E. T

Answer Option: B
Explanation:
March (31) June(30) November(30) December(31)
16th O M S T
25th R P N Q

24. How many persons have birthday between O and T?
    A. One B. Two C. Three D. Four E. Five

Answer Option: E
25. M has birthday on which date?
    A. 25th November B. 16th December C. 25th June D. 16th November E. None of these

Answer Option: E
26. Four of the following are alike in a certain way and form a group. Which of the following does not belong to that group?
    A. O B. Q C. P D. R E. T

Answer Option: C
27. Who among the following have birthday in June?
    A. R B. S C. T D. M E. N

Answer Option: D
Direction (28-30): Study the following information and answer the questions given below.
8 persons M, N, O, P, Q, R, S, and T are standing in such a way that O is 10 m in the west from N, N is 20 m South with respect to M. M is 30 m towards West with respect to Q. P is 40 m towards South with respect to Q. R is 5 m apart from S towards North. T is 20 m towards East with respect to S. R is 30 m towards West with respect to P. 28. In which direction is Q standing with respect to R?
    A.North-West B.North C.North-East D.Cannot be determined E.None of these

Answer Option: C
Explanation:
29. If one more person U is standing towards South-West with respect to P, then in which direction is T, standing with respect to U?
    A. South-West B. North-East C. North-West D. Cannot be determined E. None of these

Answer Option: D
30. What is the direction of O with respect to S?
    A. North-West B. North C. West D. Cannot be determined E.None of these

Answer Option: A Direction(31 - 35): The following question is followed by two statements named I and II. You have to decide if these statements are sufficient to answer the asked questions and mark the correct option
    A. If the data in statement I alone are sufficient to answer the question, while the data in statement II alone are not sufficient to answer the question. B. If the data in statement II alone are sufficient to answer the question, while the data in statement I alone are not sufficient to answer the question. C. If the data either in statement I alone or in statement II alone are sufficient to answer the question. D. If the data even in both statements I and II together are not sufficient to answer the question. E. If the data in both statements I and II together are necessary to answer the question.

31. Who among P, Q, R, S and T each one having different weight is the third heaviest? I. Q is heavier than R and S. T is heavier only than P. II. Only three are lighter than R. The weight of Q is 50kg which is 2kg more than R. S is heavier only than P and T
Answer Option: B
Explanation: From statement I – Q, R/S, T, P From statement II – Q (50), R (48), S, P/T
32. On which day of the week did Anu arrive? I. Her sister Tanu correctly remembers that she did not arrive on Wednesday. II. Her mother correctly mentions that she arrived before Friday but after Tuesday
Answer Option: E
Explanation: From statement I and II together Anu arrive on Thursday.
33. How is Mohan related to Faizan I. Faizan is the brother of Neelam who is the mother of Rohan. II. Mohan has two brothers and Rohan is one of them
Answer Option: D
Explanation: null Although it is clear that Mohan is either nephew or niece of Faizan as the gender of Mohan is not given. Hence, both the statements together are not sufficient to answer the question.
34. Among Puneet, Karishma, Daya, and Ritik, who is the son of Manvendra? I. Puneet and Karishma are sisters of Ritik. II. Daya is the mother of Karishma and wife of Manvendra.
Answer Option: D
Explanation: null Since, gender of Ritik is unknown, therefore we can’t answer the question using even both the statements together
35.There are five persons named Suhana, Rashmi, Radha, Ranu and Zoya standing in a row facing south. Who is standing to the immediate left of Radha?
I. Suhana is not standing on any of the extreme ends of the row. Zoya is standing to the second left of Rashmi. II. Zoya is standing in the middle of the row. Suhana is standing at third to the left of Rashmi who is at the extreme right.
Answer Option: D
Explanation: In this type of question, we should start from statement I alone and then check the statement II alone. If we are not able to answer the question from both the statements individually, merge both the statements and try to find the answer to the asked question. As, From statement IGiven that, ‘Suhana is not standing on any of the extreme ends of the row. Zoya is standing to the second left of Rashmi’. We can’t answer the question using this statement alone because there are so many possibilities for the position of Radha. From statement IIGiven that, ‘Zoya is standing in the middle of the row. Suhana is standing at third to the left of Rashmi who is at the extreme right’. Ranu/Radha Rashmi Zoya Radha/Ranu Suhana OR Rashmi Ranu/Radha Zoya Suhana Radha/Ranu We can’t answer the question using this statement alone as the position of Radha is not clear. From statement I and IIGiven that, ‘Suhana is not standing on any of the extreme ends of the row. Zoya is standing to the second left of Rashmi’ and ‘Zoya is standing in the middle of the row. Suhana is standing at third to the left of Rashmi who is at the extreme right’. ⇒ Rashmi Ranu/Radha Zoya Suhana Radha/Ranu We can’t answer the question even after using both the statements together as the position of Radha is still not clear
Direction (36-40):Study the following information to answer the given questions.
Eight people are sitting in two parallel rows containing four people each, in such a way that there is an equal distance between adjacent persons. In row 1 A, B, C and D are seated (but not necessarily in the same order) and all of them are facing south. In a row, 2 P, Q, R, and S are seated (but not necessarily in the same order) and all of them are facing north. Therefore, in the given seating arrangement each member seated in a row faces another member of the other row. R sits second to the left of the person who faces A. S is an immediate neighbor of R. Only one person sits between A and D. One of the immediate neighbors of C faces Q. B does not sit at any of the extreme ends of the line.
36. Who amongst the following sits second to the right of the person who faces P?
    A. A B. B C. C D. D E. Cannot be determined

Answer Option: B
Explanation: null
37. Four of the following five are alike in a certain way based on the given seating arrangement and thus form a group. Which is the one that does not belong to that group?
    A. C B. R C. Q D. P E. D

Answer Option: C
38. Which of the following is true regarding C?
    A. C sits second to the right of D. B. A sits on the immediate right of C. C. S faces C. D. D is an immediate neighbor of C. E. The person who faces C is an immediate neighbour of R.

Answer Option: B
39. Who amongst the following faces R?
    A. A B. B C. C D. D E. Cannot be determined

Answer Option: D
40. Who among the following faces B?
    A. P B. Q C. R D. S E. Cannot be determined

Answer Option: D
Direction (40 - 45):Study the following information to answer the given questions.
Eight persons A, B, C, D E, F, G, and H sit on the line and all of them face north direction but not necessarily in the same order. All of them like different color viz. Red, Green, Blue, Yellow, Orange, White, Black, and Violet but not necessarily in the same order. The one who likes Blue color sits second to the right of one who likes Green color. C likes orange color. A sits fourth to the left of the person who stays likes Violet color. D sits not adjacent to H. Neither A nor the person who likes Violet color sits on the extreme end of the line. B sits third to the left of F. There is only one person sits between the G, who likes Red color and the person who likes Violet color. There are two persons sit between G and the one who likes Black color. H sits immediate left of one who likes Black color. There are two persons sit between H and F, who likes white color.
41. How many persons sit between A and B?
    A.One B.Two C.Three D.Four E.None

Answer Option: A
Explanation:
B D A F G C H E
Green Yellow Blue White Red Orange Violet Black

42. D likes which of the following color?
    A.Green B.Yellow C.Black D.Blue E.None of these

Answer Option: B
43. Who among the following sits third to right of G?
    A. D B. The one who likes Violet color C. The one who likes Black color D. A E. B

Answer Option: C
44. E likes which color?
    A. Blue B. Green C. Red D. Black E. None of these

Answer Option: D
45.Who among the following sits immediate left of the person who likes White color?
    A. D B. G C. D D.The one who likes Orange color E. None of these

Answer Option: E
Direction (46 - 49): Read the questions carefully and answer the following questions. P@Q: P is to the west of Q; P is either 2 or 12 km west of Q. P#Q: P is to the east of Q; P is either 2 or 12 km east of Q. P$Q: P is to the North of Q; P is either 5 or 9 km north of Q. P&Q: P is to the south of Q; P is either 5 or 9 km south of Q. P@$Q: P is to the northwest of point Q. (Note: Distance between P and Q is not known). P&#Q: P is to the southeast of Point Q. (Note: Distance between P and Q is not known).
46. P@Q, R&#Q, Q$R. If the distance between P and R is 13 find the distance between Q and R?
    A. 12 B. 2 C. 5 D. 9 E.None of these

Answer Option:
Explanation: null
47. U @Q, U$T, T#S, R#T, which of the following cannot be the distance between S and R?
    A. 14 B. 24 C. 4 D. 16 E.None of these

Answer Option: C
Explanation: null
48. S@$T, T@Z, X&Z; X is in which direction with respect to S?
    A.North east B.South east C. South west D. North west E.None of these

Answer Option: B
Explanation:
49. S&#T, T#Q, R@$Q; What is the position of R with respect to S?
    A. North east B. North west C. North D. South E. South west

Answer Option: B
Explanation: null
Direction (50 - 52): Study the information given below and answer the questions based on it.
Six persons A, B, C, D, E, and F have different weights and heights. B is taller than A and E but lighter than A and E. D is taller than B. E is only taller than C and F. Only one person lighter than A. D is heavier than C and E but not the heaviest. The number of people is heavier than F is same as shorter than him. Not more than two persons are heavier than C.
50. Who among the following is shortest in the group?
    A. C B. F C. E D. A E. D

Answer Option: B
Explanation:
Rank Height Weight
1(Heighest) D F
2 B D
3 A C
4 E E
5 C A
6(lowest) F B

51. Who among the following is heaviest in the group?
    A. D B. C C. F D. B E. E

Answer Option: C
Explanation:
52. How many persons are lighter than C?
    A.None B.One C.Two D.Three E.Four

Answer Option: D
Explanation:
Direction (53 - 56): Read the following information and answer the questions.
A cricket match has to be organized in two areas – Rajnagar and Mirgarh. There are four batsmen- A, B, C and D where D is a Keeper batsman and there are five bowlers- P, Q, R, S and T where P is a special keeper and S is the only spinner among them. Selection of players has to be decided by following conditions Prize Money of Players: A - 10 Lakhs P - 9 Lakhs B - 9 Lakhs Q - 7 Lakhs C - 8 Lakhs R - 8 Lakhs D - 7 Lakhs S - 10 Lakhs T - 6 Lakhs
1) There should always be a keeper behind the wickets.
2) The budget of Rajnagar is not more than 55 lakhs and the budget of Mirgarh is not more than 60 lakhs.
3) The pitch of Rajnagar supports fast bowlers so all fast bowlers must play in this match and wicket falls too quickly so more than two batsmen are required.
4) The spinner must play in the team of Mirgarh and more than two batsmen and at least two bowlers are playing in this team.
5) Those players who do not play in the first match must play in the second match.
6) The match rules are unconventional and a number of players may vary in each match.
53. Who among the following must play in the match of Rajnagar?
    A.D and P B.B and S C.D and P D.D and T E.None of these

Answer Option: D
Explanation: The budget of Rajnagar is not more than 55 lakhs and all fast bowlers must be included in the team. So, Q, R and T are selected in this team. Q+ R+ T = 7 + 8 + 6 = 21 lakhs At least two batsmen are selected and a keeper must include. So, suppose A, B and C are selected. Total budget = A+ B+ C = 10+ 9+ 8 = 27 lakhs. Between D and P one will be selected. 21+ 27 = 48 lakhs For D, budget = 48+ 7 = 55 lakhs. For P, budget = 48+ 9 = 57 lakhs So final selected players are Q, R, T, A, B, C and D, and T must be selected.
54. P must play with which player?
    A. B B. D C. R D. T E. S

Answer Option: E
Explanation: As P and S are not selected in the first match they must play in the second match so P must play along with S.
55. If budget of Rajnagar lowers down to 45 lakhs, then which of the following player is not playing in this match?
    A. Q B. R C. D D. A E. B

Answer Option: D
Explanation: Based on the conditions two teams are possible:- First team- Q, R, T, D, B and C = 45 lakhs Second team- Q, R, T, P, C and D = 45 lakhs So player A is not playing in this team.
56. If all the batsmen must include in team of Mirgarh, then which of the following player will not play for mthis team?
    A. Q B. R C. T D. S E.None of these

Answer Option: B
Explanation: As all the batsmen are playing along with them P and S are also playing as they are not playing in the first match. A+ B+ C+ D+ P+ S = 53 lakhs Total budget of Mirgarh is not more than 60 lakhs. So R is not playing in this team. Direction: In each of the following question, two statements numbered I and II are given. There may be a cause and effect relationship between the two statements. This two statement may be the effect of the same cause or independent causes. These statements may be independent causes without having any relationship. Read both the statements and decide which of the following answer choice correctly depicts the relationship between these two statements. I. The standard of living in the Indian setup has seen a sharp rise along with with a marked increase in consumer spending over the past few years. II. The gross domestic income has seen a fluctuation in the past few years which might not bode well for the Indian economy at large 57.
    A. Statement 1 is a cause and statement 2 is an effect. B. Statement II is the cause and statement I is its effect. C. Both the statements I and II are independent causes. D. Both the statements I and II are effects of independent causes. E. Both the statements I and II are effects of some common cause.

Answer Option: D
Explanation: Neither statement has seen any corroboration for why it is happening making option [d] the best suited option.
Direction (58 - 60): Read the following information carefully and answer the following question:
You have to assign 1- 8 numbers to the consonant letter. For example, B is assigned to 1, C is assigned 2 up to the K is assigned 8. Again start from L assign to 1 and so on. Following are the rules, apply in the letter according to the given condition and coded accordingly:
i) If a word contains vowel in starting and ending both ends then replace both vowels with * and # respectively.
ii) If a word contains 3 or more vowels then first two vowels are replaced with % and * and opposite another vowel.
iii) If there are immediate vowels then interchange its position and opposite the letters and increases its letter coded value by 2 numbers
iv) If the first letter is a vowel and the last letter is consonant than interchange its position and opposite vowel letter.
v) If there are two vowels in the word but not immediate then interchange its position and opposite both letter.
58.The code for the word ‘philatelist’?
    A. 46%1*821768 B. 46%1*812678 C. 46%1*821678 D. 46%1*811678 E.None of these

Answer Option: D
Explanation: B=1, C=2, D=3, F=4, G=5, H=6, J=7, K=8 L=1, M=2, N=3, P=4, Q=5, R=6, S=7, T=8 V=1, W=2, X=3, Y=4, Z=5 ‘P h i l a t e l i s t’ By applying condition 2 here: 46%1*8V1R78 → 46%1*811678
59.Which of the following is the code for ‘MUSIC BITE’?
    A. 26642 1266 B. 26732 1176 C. 26742 1186 D. 26632 1286 E.None of these

Answer Option: C
Explanation: B=1, C=2, D=3, F=4, G=5, H=6, J=7, K=8 L=1, M=2, N=3, P=4, Q=5, R=6, S=7, T=8 V=1, W=2, X=3, Y=4, Z=5 ‘MUSIC BITE’ ‘MUSIC’ By applying condition 5 ‘M U S I C’ → M I S U C 2 R 7 F 2 → 26742 ‘BITE’ By applying condition 5 ‘B I T E’ → B E T I 1 V 8 R → 11 86
60. Addition of all independent numbers in the word ‘your fairy tales’?
    A. 63 B. 64 C. 65 D. 66 E.None of these

Answer Option: A
Explanation: B=1, C=2, D=3, F=4, G=5, H=6, J=7, K=8 L=1, M=2, N=3, P=4, Q=5, R=6, S=7, T=8 V=1, W=2, X=3, Y=4, Z=5 ‘your fairy tales’ ‘Y o u r’→ 4 o u 6 By applying condition 3 4 o u 6 → 4 u o 6 → 4 l f 6 → 4146 → 4366 = 19 ‘F a i r y’ → 4 a i 6 4 By applying condition 3 4 a i 6 4 → 4 i a 6 4 → 4rz64 → 46564 → 48764 = 29 ‘T a l e s’ → 8 a 1 e 7 By applying condition 5 8 a 1 e 7 → 8 e 1 a 7 → 8 v 1 z 7 → 8115 = 15 19 + 29 + 15 = 63

Direction (1-5): Read the following passage carefully and answer the questions that follow.
Beijing now produces almost 15 billion pounds of municipal household waste each year; the landfill at Asuwei receives 7.2 million pounds. In an effort to deal with the growing amount of trash generated by China’s expanding middle class and increasingly consumerist society, Beijing and other cities are building huge incinerators—including one close to the landfill in Asuwei. The people are worried; once fully operational, the new incinerator will reportedly burn 6 million pounds of garbage a day. Environmental authorities have said the project passed the required assessments, but the residents are mistrustful, fearing the emissions that will come from the plant. “The environment cannot tolerate it, and our health would be at risk,” a lady says. She maintains that everyone in the area is against the incinerator, though only a few— including herself—are taking action, organizing and participating in public protests. She spends a lot of time reading and analyzing reports and environmental assessments, trying to find a way to stop the incinerator from opening. China's leaders appear fearful that the many, small localized bands of discontented citizens will coalesce into a larger movement; it now spends more money on internal security, which includes managing and suppressing these protests, than it does on its military. The unification of what is now disconnected grassroots actions against specific pollution sources into a national environmental movement is perceived as a threat to the rule of the Communist Party. The Internet, used by activists and protesters as a tool for sharing information, is often quickly scrubbed of evidence of any protest actions or criticism of the government. Earlier this year, a documentary film on China's environmental ills received hundreds of millions of hits in just one week before being taken down by government censors, presumably out of fear that it could become China's Silent Spring moment, sparking a nationwide outcry. At the same time, citizens are fearful too—of the rash of toxins that threaten their lives, and of the government that has shown it is willing to punish those who dare complain about the threat. Nevertheless, complaining they are, in increasing numbers and with increasing boldness— and impact
1. Which of the following best describes the central idea of the passage?
    A. Growing economy of the country B. Banning of the Internet by the Government to stop interaction C. Use of harmful chemicals by the Government plants D. Extension of Government support to the harmful chemical plants E. Ignorance and suppression of the growing environmental threat by the Government

Answer Option: E
Explanation: The passage states how pollution and the associated environmental problems are increasing in the country but the Government has chosen the easy and the wrong way. It is trying to curb all sorts of protests and steps taken by the citizens and social unrest exists. The passage states the negligence that exists from the Government’s side. Thus option E is the correct answer
2. Which of the following can be inferred from the passage?
    A. The industrialization process must be stopped in order to protect the environment B. The Government harasses the people who try to create awareness about the environment and engage in protests. C. The Government is trying its best to curb pollution but it will take time to be effective. D. The Government does not want to harm the citizens but it will take time to cut down on the rate of pollution. E. The Government does not want to create unemployment problems thus cannot close down certain plants.

Answer Option: B
Explanation: The passage states how the people have started protesting against the negligent steps taken by the Government. If anyone tries to create awareness the Government takes steps and often bribes people to stop such movements. Thus option B is the correct answer.
3. Which of the following is the major issue discussed in the passage?
    A.The Government is supporting the growth of certain chemical plants which should be shut down keeping in mind the environmental issues. B. The Government wants to suppress all the voices which have pointed out the fallacies in the economic policies. C. The Government is treating the waste materials in a deleterious way that threatens the health of people. D. The Government is practicing the use of nuclear weapons on the citizens which has caused havoc in places. E. The Government has chosen to stop all efforts for the cause of environmental up-gradation.

Answer Option: C
Explanation: The passage throws light on how incinerators are being used to treat the wastes dumped in huge amounts. The smoke is harmful to the health of people. However, any sort of environmental concern is being ignored by the Government.
4. Which of the following can be inferred from the given passage?
    A. The citizens dare not complain against the Government as the number of people protesting is really less. B. The Government has taken some initiatives to curb the pollution but they are not satisfactory. C. The citizens are more concerned about losing their jobs so would not protest much. D. The Government is too powerful to be complained against, hence no efforts are likely to Nbring about any change. E. None of these

Answer Option: D
Explanation: An inference is something which is not explicitly stated. The last line of the passage indicates that the citizens are complaining that too in large numbers and are continuing their protests against the Government. Thus option E is the correct answer.
5. Which of the following can be inferred correctly from the line ‘Earlier this year, a documentary film on China's environmental ills received hundreds of millions of hits in just one week’?
    A. Instigating people against the government is on the process. B. The use of the Internet is high and any topic can be brought to the notice of the public through the Internet. C. The citizens of China are aware of the environmental degradation and have the mindset, to protest against it. D. Film making is a powerful force or tool that helps bring forth any issue. E. The environment is seriously deteriorating in the country and is not a made-up story

Answer Option: C
Explanation: It is clear from the passage that people are aware of the degradation of the environment and the Government is at its best to curb all these protests and hide the damage that is being done to the environment. The main point that is significant here is the awareness of the people and their intention to do something about it. Thus C is the correct answer
Direction(6 - 10): In the given question, a part of the sentence is printed in bold. Below the sentence alternatives to the bold part are given at (A), (B), (C) and (D) which may help improve the sentence. Choose the correct alternative. In case the given sentence is correct, your answer is (E) i.e. No correction required.
6. What drove its initial development was its purely digital existence, away from the control of government regulators
    A. away to the control of B. away with control of the C. away from control of the D. away with the control for E. No correction required

Answer Option: E
7. Bitcoin could let transacting party remain anonymous, keep transactions very secure, and eliminate middlemen fees
    A.would letting transactions parties remains B.should lets transacting party remain C.would let transacting parties remain D.would have transacting parties remain E.No correction required

Answer Option: C
Explanation: 'Would let transacting parties remain' is the correct answer. 'Would' is used to preserve the future aspect when talking about the past. If you are writing about past events, you can use it to indicate something that was in the future at that point in time but is not necessarily in the future right now. 'Could' represents past of 'can' and when you want to convey something politely. Here, the future aspect of Bitcoin is being discussed, hence would is a more suitable response. Would letting is grammatically incorrect; hence option A can be eliminated. Option D makes the sentence grammatically incorrect; hence it can be eliminated.
8. The mid-day meal scheme has helped but should it have been linked less with the schooling system and more with the hunger pattern
    A.should had been linked less B.. should it not have been linked less C.t should been linked lesser D.it should have been linked less E.No correction required

Answer Option: D
Explanation: The correct structure for the given sentence is affirmative. Hence, 'it' must be placed before 'should'. This eliminates the given sentence and option B. The given sentence is generic; hence had is an incorrect form. Hence, option A can be eliminated. Option C is incomplete. Option D completes the sentence well.
9. A true leader's life is a saga of dedicating to the cause of welfare of human beings
    A.of dedicated B.of having dedication in C.of dedication to D.for dedication at E.No correction required

Answer Option: C
Explanation: A noun is required after 'a saga of'. Dedicating is the gerund form (verb). Option A: 'dedicated the cause of' is incorrect. Option B: Having before dedication distorts the structure. Option C: It fits the given sentence well. Option D: The correct proposition after saga should be 'of' and not 'for'.
10. The building had been under construction for a long time until a series of unprecedented events led it to a halt.
    A. Leads it to a halt B. bringing it to a stop C. brought it to a halt D. led it to a stop E. No correction required

Answer Option: C
Explanation: The answer to this is the statement 'C.' The correct sentence is “The building had been under construction for a long time until a series of unprecedented events brought it to a halt.” Bring something to halt is a phrase that means causing something to stop. According to the sentence, some unprecedented events stopped the construction of the building. Thus the phrase (in its past tense) 'Brought it to a halt' is apt for this sentence.
Direction(11 - 16): Read the given passage carefully and answer the questions that follow. Certain words are printed in bold to help you locate them while answering
some of these. Advice about the art of interview preparation and how to craft the perfect CV isn’t enough to put every student on a path to a career they want. About one in three graduates end up being “mismatched” to the jobs they find after leaving university, research by Universities UK suggests. These mismatched graduates face poorer prospects and lower earnings than their peers who embark on careers that are a better fit for the knowledge and skills they have acquired through three or four years of study. It suggests that traditional careers advice isn’t working. The problem isn’t necessarily that too many students are taking the wrong course. There is little evidence that graduates are studying the “wrong” subjects, according to the UUK research, since most are on courses that offer subject knowledge and employability skills that are very much in demand. Instead, students need better careers advice that will help them define their skills and attributes – and understand how these match different career options. Students also need help finding out which skills they’ll need to break into certain industries – particularly in sectors that aren’t good at diversifying their recruitment, or when they have no family or social network of contacts to call on for help and advice. Politicians complain of a skills gap, but graduates face an “experience gap” – with many employers preferring to recruit young people who have spent a couple of years in the workplace rather than raw recruitment from the university. To help graduates find the right jobs for them, lots of universities are experimenting with new ways to make their careers advice more accessible and meaningful. At the University of Kent, students can use an online Careers Explorer service to match their skills to career options, and a work-study scheme that provides bursaries for work experience. Students at the University of Dundee can take employability modules in parallel with their academic work, including online and personal career planning sessions.
11. Which of the following statements is/ are true with respect to the passage? I. The graduates who do not end up getting a job best suited to their knowledge and skill end up earning less than their counterparts who pursue their skills. II. A lot of universities have come forward to address the problem of mismatched jobs. III. The lack of good pieces of career advice causes graduates to enter into professions that are not in sync with their skill and profession.
    A. Only I B. Only II C. Both I and II D. Both II and III E. All I, II and III

Answer Option:
Explanation:
12. Which of the following myths does the author break in the passage?
    A. The skills acquired by graduates is not equivalent to the degree they have achieved. B. Instead of a wrong choice of an academic course, the problem lies in the lack of provision of a good career advice, which is well- matched with the skill and knowledge of a graduate. C. The fact that the courses available at universities are not skill oriented, pushes the graduates to be caught up in wrong jobs. D. The career advice being provided at the universities do not effectively analyse the potential of a graduate and provide them with good career option. E. None of these.

Answer Option: B
Explanation: The third and the fourth paragraphs of the passage suggest that students are not opting for the wrong courses, but they have not been counseled appropriately to get the job that matches their knowledge and skills
13. With which of the following statements is the author most likely to agree?
    A. Most of the students often enroll in courses that do not offer subject knowledge and employability skills that are in demand. B. The initiatives being taken by the Universities to provide accessible career options may not be successful. C. The traditional careers advice would work if they are directed towards skill development rather than imparting theoretical knowledge. D. The employers have a tendency to recruit recruits from Universities as they can be hired at significantly low remunerations. E. Students require help in finding the skills required to get into employment sectors at a time when they do not have access to good careers advice through their family and peers.

Answer Option: E
Explanation: The author of the passage supports the university graduates need to be provided professional advice with respect to the career they must opt for. This would save them from making unsuitable career choices. According to the passage, "Students also need help finding out which skills they’ll need to break into certain industries – particularly in sectors that aren’t good at diversifying their recruitment, or when they have no family or social network of contacts to call on for help and advice."
14. With reference to the context of the passage, what can be inferred from the line, “Politicians complain of a skills gap, but graduates face an “experience gap”?
    A. The politicians have not been able to properly analyze the root cause of the problem that lands up a student in an undesirable job. B. The politicians have rightfully identified the major cause of the unemployment problem. C. The problem of “experience gap” is identified and well- addressed by the politicians. D. The priority of the politicians is disoriented as they emphasize more on experience than on skills. E. None of these.

Answer Option: A
Explanation: The author of the passage points out that many employers prefer recruiting young people who have spent a couple of years in the workplace rather than raw recruitment from the university. This implies that the problem lies in the lack of experience as opposed to the lack of skills, something which the politicians complain of. Thus the mentioned sentence implies that the politicians have not been able to properly analyze the root cause of the problem that lands up a student in an undesirable job.
15. Which of the following words is MOST SIMILAR in meaning to the "evidence" as given in the passage?
    A. Result B. Proof C. Feature D. Rustic E. Misnomer

Answer Option: B
Explanation: "Evidence" means proof. Thus, option B is the correct answer. Rustic- made in a plain and simple fashion. Misnomer- a wrong or inaccurate use of a name or term.
16. Which of the following words is MOST OPPOSITE in meaning to "embark" as given in the passage?
    A. Undertake B. Process C. Reject D. Apprehend E. Deceive

Answer Option: C
Explanation: "Embark" means to begin (a course of action). Thus, option A is the synonym of "embark". "Reject" is the correct antonym of the given word Apprehend- understand or perceive.
Direction(17 - 20): In the following question, a word is given that is followed by three statements. All these three statements carry the given word. Identify the sentence(s) in which the word fits to make them grammatically correct and meaningful.
17. Hamper I. The Center's move to direct oil marketing companies to lower auto fuel price hampers the confidence of oil companies in investments made in India. II. Metro Railway services were hampered for 15 minutes on Monday afternoon. III. High street retailers, Marks & Spencer and John Lewis & Partners, have also released their own hampers which are far more affordable than the luxury offerings.
    A. Only I B. Both I and II C. Only III D. Both II and III E. All I, II and III

Answer Option: E
Explanation: All three statements use the word “hamper” in the correct form. In the first and the second sentence, the word has been used as a verb in the present and the past tense respectively. “Hamper” means to cause hindrance. The third sentence uses the word as a noun which means a basket or a container. The word fits appropriately in the given question. Since the word has been used correctly in all the sentences
18. Tact I. The kind judge was tactful when he informed the dancer of her elimination from the show. II. They came up with a tactfully to reach their destination in the fastest time possible. III. He had an engaging personality and used tact in dealing with his patients.
    A. Only I B. Only III C. Both I and II D. Both I and III E. All I, II and III

Answer Option: D
Explanation: Tact— skill, and sensitivity in dealing with others or with difficult issues Tactfully— with skill and sensitivity in dealing with others or with difficult issues Tactful— having or showing skill and sensitivity in dealing with others or with difficult issues “Tactful” fits in statement I as an adjective is required to define the judge. “Tact” fits in statement III as it has been correctly used as a noun in the sentence. “Tactfully” is an adverb which can modify a verb, an adjective or another adverb. The word does not fit in the second sentence as one cannot come up with a “tactfully”, but with tact. Since only statements I and III are correct
19. Adage I. Despite what that old adage says, I like to have my cake, eat it, and then I have another piece. II. Among other things, the writer is famous for managing his philosophical ideals. III. Tired of all the same old sayings, he adages a new thought.
    A. Only I B. Only II C. Only III D. Both I and II E. Both II and III

Answer Option: A
Explanation: “Adage” is a noun which means a proverb or short statement expressing a general truth. The word cannot be used in the verb or a gerund form, thus, statements II and III are incorrect. The word has been used correctly only in statement I.
20. Malaise I. After his dog died, he remained malaise for several months. II. Many citizens who live near the contaminated river are complaining of a malaise that keeps them bedridden. III. Weeks before his heart attack, Mr Robbin kept mentioning how he felt malaiseful and not like his normal self.
    A. Only I B. Only II C. Both I and III D. Both II and III E. All I, II and III

Answer Option: B
Explanation: “Malaise” (noun) means uneasiness or restlessness. The word has been correctly used only in statement II. The other two sentences use the forms of the word which do not exist in the Standard English language Direction(21-25): In the following questions, two columns I and II, each containing three sentences are given. Column I consists of the first part of three sentences and Column II consists of the remaining part of those three sentences. Match column I with column II, so that the sentences formed are meaningful and grammatically correct.
21. I. A) We believe in a circular rather B) The Rig Veda was written more than 3800 years ago, C) The word 'Juggernaut' was originally used to denote the Rath Yatra temple car, II. D) Veda is superior to Albert Einstein's Theory of Relativity equation. E) than a linear concept of time. F) which was so massive it would crush devotees under the wheels.
    A. B-E and C-F B. A-E, B-F and C-D C. A-F and C-D D. B-F E. A-E and C-F

Answer Option: E
Explanation: When we use 'rather', we mention the opposite of that action in the next phrase. Since circular and linear are opposite, A & E go together. C and F also connect appropriately.
22. I. A) A garbage patch in the Pacific Ocean B) There's a mass of floating rubbish in the Pacific Ocean weighing around C) About 80 percent of the plastic trash that makes up the Great Pacific Garbage Patch II. D) is believed to have originated from land-based activities. E) to help farmers who depend on the weather. F) has stretched to almost twice the size of France.
    A. B-E and C-F B. A-E, B-F and C-D C. A-F and C-D D. B-F E. A-E and C-F

Answer Option: C
Explanation: All the given sentences are about oceans. Nothing is related to farmers. B ends with 'around' which indicates there must be a location mentioned in the next part of the statement. None of the segments in column II does that. Both D & F can follow A. But none of the options mentions A- D as a pair. So, D will follow C and F will follow A.
23. I. A) Overfishing can wreak havoc to marine ecology and completely B) Unsustainable fishing practices over the last few decades have pushed C) Oil destroys the insulating and water-repellent properties of II. D) eventually accumulating in remote areas of the world’s oceans. E) our oceans to a point where they may now be on the verge of collapse. F) the marine animals, exposing them to harsh environment.
    A. B-E and C-F B. A-E, B-F and C-D C. A-F and C-D D. B-F E. A-E and C-F

Answer Option: A
Explanation: Statement A ends with 'completely', an adverb. This means the part following it must start with a main verb. None of the clauses in column II do so. Statement C ends with 'of' which indicates that the next part must describe an attribute of the subject (Unsustainable fishing practices). Grammatically, both E & F can do that. But, none of the options has C- E as a pair. So, F would be more appropriate after C. B & E can be joined correctly. Hence, the correct answer
24. I. A) At Harvard, Sheryl founded a group meant to encourage women B) Sheryl Sandburg is responsible for spearheading several successful Facebook projects C) In the hall of fame of women breaking the glass ceiling and painting their name on it, II. D) and ended up becoming the first Indian woman to obtain a degree in western medicine in the late 1800s. E) to know so much about them as possible. F) that have made the social networking site the market giant it is today.
    A. B-E and C-F B. A-E, B-F and C-D C. A-F and C-D D. B-F E. A-E and C-F

Answer Option: D
Explanation: There is only one 'social media market giant' in column I, i.e. is Facebook. So, B goes with F. No other combinations can be formed correctly. Hence, the correct answer is option D.
25. I A) Elephants prefer one tusk over the other, B) Elephants use their feet to listen, they can pick up C) Elephants herds are matriarchal, with older females taking II. D) care of the calves and protecting them while traveling from place to place. E) just as humans are mostly either left or right-handed. F) sub-sonic rumblings made by other elephants, through vibrations in the ground.
    A. B-E and C-F B. A-E, B-F and C-D C. A-F and C-D D. B-F E. A-E and C-F

Answer Option: B
Explanation: Care of calves and humans cannot be picked up for listening. So, B goes with F. Preferring one appendage over another, is a habit also see, in humans i.e. being right or left-handed. So, A & E go together. Older females do care of the young ones. so, C goes with D.
Direction(26 - 30): In the given question, a part of the sentence is printed in bold. Below the sentence, three alternatives to the bold part are given which may help improve the sentence. Choose the option that reflects the correct use of the phrase in the context of the sentence. In case the given sentence is correct, your answer is (E), i.e., "No correction required".
26. No other region in the world illustrates the chronic nature of displacing caused by extreme weather events and climate change more than Asia and the Pacific. i. Chronic nature of displacement caused by extreme weather. ii. Chronic nature of displacement caused by extreme weather. iii. Chronic nature of displacing caused by extreme weather.
    A. Only i B. Only ii C. Only iii D. Both ii and iii E. No correction required

Answer Option: A
Explanation: "Displacing" is a continuous verb which does not make much sense here. Thus, the noun "displacement" should be written instead of "displacing". Also, the sentence talks about climatic changes, thus, the word "weather" should be used instead of "whether". Since only (i) is correct, thus, option A is the correct answer. Note that "weather" (alternative i) and whether (alternative ii) are two different words carrying different meanings.
27. The recent incidents of drug overdose-related deaths have brought the spotlight back in the drug menace in the state and on the role of the Punjab Police in curbing it. i. Overdosing-related deaths have brought the spotlight back over the drug ii. Overdose-related deaths have brought the spotlight back on the drug iii. Overdose-related deaths have brought the spotlight back on the drug
    A. Only i B. Only ii C. Only iii D. Both ii and iii E. No correction required

Answer Option: B
Explanation: The subject here is "incidents" which is plural, thus, the verb should be plural too. Hence, "have" should be used instead of "has". Moreover, 'spotlight in' will get replaced by the 'spotlight on' as there should be a proper use of the preposition. 'To put the spotlight on' something means to highlight it
28. More than anything, it is the human cost of disasters that are the most compelling argument for action. i. the human cost of disasters that the most compelling ii. the human cost of disasters that is the more compelling iii. the human cost of disasters that is the most compelling
    A. Only i B. Only ii C. Only iii D. Both ii and iii E. No correction required

Answer Option: C
Explanation: "Cost" is a singular subject, thus the auxiliary verb "is" should be used. Thus, option C is the correct answer. "More" represents comparative degree, hence, "the" cannot be used before it, hence, (ii) is incorrect
29. Our Integrated Child Development Services, meant to provide nutrition and childcare up to six year of age, lack greatly in qualitatively and coverage. i. years of age, lack greatly in quality and coverage ii. years of age, lack greatly in qualitatively and coverage iii. year of age, lack greatly in quality and coverage
    A. Only i B. Only ii C. Only iii D. Both i and ii E. No correction required

Answer Option: A
Explanation: "Six" is a plural number, thus, 'years' should be in the plural as the preposition "of" is used after it. In contrast, in compound nouns, the singular forms are used. E.G. I saw a ten-year-old boy steal from the shop. Additionally, in the highlighted part, the phrase "in qualitatively" is incorrect as "qualitatively" is an adverb. To make the sentence correct, the noun "quality" should be used. Since (i) makes both the corrections
30. The role of meritocracy in Chinese history focused on the so-called\ examining system, have been a matter of long scholarly scrutiny. i. focused on the so-called examination system, has ii. focused on the so-said examination system, has iii. focuses on the so-called examine system, have
    A. Only i B. Only ii C. Only iii D. Both ii and iii E. No correction required

Answer Option: A
Explanation: The subject here is "meritocracy", hence, the singular verb "has" should be used instead of "have". "Examing" as given in alternative (iii) is not a word in Standard English, hence incorrect. Thus, option A is the answer.
Direction(31- 35): In the question, one sentence is given, and four words have been given in bold denoted by (A), (B), (C) and (D). You have to decide which of the following is inappropriate in the context. If all the words are appropriate in the context then mark ‘All correct’ as your answer.
31.Indigestion (A) is often a sign of an underline (B) problem, such as an ulcer (C), rather than a condition (D) on its own.
    A. A B. B C. C D. D E. All are correct

Answer Option: B
Explanation: The error is in part B of the sentence, which means the word “underline” has been used wrongly here. Note that the word mentioned after “underline” is “problem”, which is a noun. So, we need an adjective to modify this noun. Thus, “underlined” should replace the highlighted word mentioned in B. An underlined problem means an emphasised problem
32. A microwave heats food by causing (A) the molecules to vertebrate (B) but it certainly does not make food radioactive, (C) as a misconception (D) suggests.
    A. A B. B C. C D. D E. All are correct

Answer Option: B
Explanation: The error is in part B of the sentence, which means the word “vertebrate” has been used wrongly here. Note that the sentence mentions that the heat causes some action in the molecules, which means we need a verb instead of the noun “vertebrate” (an animal of a large group distinguished by the possession of a backbone or spinal column). Thus, the word “vibrate” must be used which means move continuously and rapidly to and fro.
33. She saw a fleet (A)/ of ships (B)/ in the nearby (C)/ sea shore. (D)
    A. 1 B. 2 C. 3 D. 4 E. All are correct

Answer Option: B
Explanation: Fleet is the collective noun used for ships. Here, 'ships' has been used in the first part, which needs to be replaced with 'ships'
34. The prices of electronic goods have declined (A)/ dew to the (B)/ reduction in (C)/ import duty rates. (D)
    A. A B. B C. C D. D E. All are correct

Answer Option: B
Explanation: Dew should be replaced by due. Dew means tiny drops of water that form on cool surfaces at night when atmospheric vapor condenses. Due to means owing to something or because of something.
35. By created (A)/ the most precise (B)/ lunar gravity (C)/ map, scientists (D) hope to find out what is beneath the lunar surface.
    A. A B. B C. C D. D E. All are correct

Answer Option: A
Explanation: Here "creating" should be used in place of "created" in order to make sentence context appropriate.
Direction(36 - 40): In the given question, a statement has been divided into five segments, each of which is denoted by (A), (B), (C), (D) and (E). Rearrange all the segments to form a coherent statement.
36. A) however, the UPSC decided to B) cheating, the normal C) consequence is his disqualification, D) if an able-bodied student engages in E) get the guidelines changed
    A. DBCEA B. DBCAE C. CABDE D. ABCDE E. ABECD

Answer Option: B
Explanation: The usage of “however” in one of the segments suggests that a contradiction to a mentioned clause will be presented in the other clause. The sentence cannot start with the conjunction, "however". This eliminates options D and E. Part C cannot begin the sentence either as it would fail to make a logical sentence. E cannot immediately follow C, thus, the correct sequence
37. A) Harry and Meghan's little one B) will be seventh in line to the throne C) when he/she arrives and will be D) about a year younger than E) the reigning baby of the family
    A. ACBDE B. ABCDE C. ACEDB D. BDECA E. CEBAD

Answer Option: B
Explanation: The sentence should start with the main subject, which in this case is "Harry and Meghan's little one". The little one's objective is given by segment B and hence that should follow A. Of all the options, it is option B which has AB as the opening pair. Option B is the correct answer and the correct sequence is ABCDE
38. A) a brief career in music, where they performed B) in concerts and as standalone artists to C) the couple moved to Mumbai to have D) after the wedding, E) mesmerized the entire country.
    A. ABCDE B. CDABE C. None of these D. BDCEA E. AEBCD

Answer Option: C
Explanation: The segments cannot be arranged in any of the given sequences as there are grammatical errors in it.
39. A) the adjective “resilient” and the nouns B) of disasters and emergencies who recover C) quickly and from adversities D) “resilience” and “resiliency” are terms E) that are normally applied to the victims
    A. ACEBD B. EBCDA C. CDEAB D. ADEBC E. ADBCE

Answer Option: D
Explanation: The theme of the sentence is centered around the given words, “resilient” “resilience” and “resiliency”. Thus, A and D can be used to start the sentence. Of the two, A must be followed by D. Now E tells us about the use of the words, which is applied to victims. Thus, the sequence so far is ADE. The option with the same sequence is D and the correct sequence is ADEBC
40. A) Frontwoman Lauren May berry B) complimented the blue neon C) lights illuminating the band D) mirrored dress that E) took the stage in a pastel-colored
    A. ABCDE B. BDACE C. CBDAE D. ECADB E. AEDBC

Answer Option: E
Explanation: The sentence is about Lauren May berry. Thus, segment A should start the sentence. Frontwoman means the leading singer in a band, thus, logically she got on to the stage must follow A. She took the stage in something that was pastel-colored. This inadvertently means that she was dressed up in a dress which was pastel in color. What must follow E is D. The option with the sequence AED is option E and AEDBC is the correct order.
Other Articles

shape Job-Alerts

Competitive Exams - Recent Job Notifications
Category
Banking SSC Railway
Defence Police Insurance
Click Here For – All India Latest Jobs

shape SP Quiz

Competitive Exams - Practice Sets
Category Quiz
Quant Aptitude Simple Interest
Spotting Errors
Reasoning Ability Puzzles
Seating Arrangement
Insurance Awareness Insurance Awareness

shape GK


Nainital Bank Recruitment Notification – PO & SO - Related Information
Nainital Bank Recruitment Notification – PO & SO